RD obstetrics Flashcards

1
Q
  1. Adnexal mass (? Ill-defined), heterogenous echogenicity on US (? Has echoes), isointense on T1 and homogenously dark on T2. Which of the following is most likely:
    a. Hemorrhagic cyst
    b. Simple cyst
    c. Endometrioma
    d. Mature teratoma
A

c. Endometrioma T at US contains uniform low-level echoes (ground glass); at MRI high T1 signal that persists with FS; T2 signal varies, but characteristic is low T2 signal (T2 shading)
1. Adnexal mass (? Ill-defined), heterogenous echogenicity on US (? Has echoes), isointense on T1 and homogenously dark on T2. Which of the following is most likely:
a. Hemorrhagic cyst F = may have intermediate-high T1 signal secondary to haemorrhage, without loss of signal on fat sat images; usually high T2 signal however, as opposed to endometriomas which often have T2 low signal (“shading”)
b. Simple cyst F not ill-defined, no echoes if simple
c. Endometrioma T at US contains uniform low-level echoes (ground glass); at MRI high T1 signal that persists with FS; T2 signal varies, but characteristic is low T2 signal (T2 shading)
d. Mature teratoma F unlikely to be of homogeneous low T2 signal – exception would be fibroma/fibrothecoma

How well did you know this?
1
Not at all
2
3
4
5
Perfectly
2
Q
  1. Young woman (20 something), 6 weeks after last period, pelvic pain for several hours. Presents to ED. β-hcg pending. US shows (hyperechoic) fluid in pelvis and blurred uterine outline with hyperechoic adnexa. Most likely?
    a. Appendicitis
    b. Ruptured ectopic
    c. Endometrioma
    d. Ovarian torsion
    e. Tubo-ovarian abscess
    f. Mucinous tumour
A

*LW:
According to statDx and Promethius: “indefinite” uterus sign = PID.
“Indefinite” uterus sign
Inflammation and echogenic fluid in the cul-de-sac obscure uterine margins, especially along posterior border (also seen on CT).
Thus favoured option is tubo ovarian abscess.

Previous answer:
Ruptured ectopic

  1. Young woman (20 something), 6 weeks after last period, pelvic pain for several hours. Presents to ED. β-hcg pending. US shows (hyperechoic) fluid in pelvis and blurred uterine outline with hyperechoic adnexa. Most likely?
    a. Appendicitis F

b. Ruptured ectopic F = uterus usually normal appearance, adnexal mass but ovary often seen as separate 6 weeks post period with hyperechoic fluid – she pregnant dawg
c. Endometrioma F
d. Ovarian torsion F
e. Tubo-ovarian abscess T cystic/solid/heterogenous adnexal mass, normal ovarian morphology not recognisable (Zagoria p274); uterus enlarged, ill-defined Again, pregnant.
f. Mucinous tumour F

How well did you know this?
1
Not at all
2
3
4
5
Perfectly
3
Q
  1. Antenatal scan shows anterior abdominal wall mass with cord inserted into side.
    a. Omphalocele
    b. Pseudo-omphalocele
    c. Gastroschisis
    d. Cord AVM
A

c. Gastroschisis T if cord inserts into abdominal wall adjacent to mass (Callen p627, Pocket p131)
3. Antenatal scan shows anterior abdominal wall mass with cord inserted into side (LAS – cord inserts into abdo adjacent to mass).
a. Omphalocele T if cord inserts onto mass
b. Pseudo-omphalocele F = deformation of the fetal abdomen by transducer pressure coupled with an oblique scan orientation
c. Gastroschisis T if cord inserts into abdominal wall adjacent to mass (Callen p627, Pocket p131)
d. Cord AVM F mass will be within cord

How well did you know this?
1
Not at all
2
3
4
5
Perfectly
4
Q
  1. Antenatal scan at 20 weeks shows placenta over os. Technician asks you to check, and placenta is now clear of os. Which is most likely?
    a. Braxton Hicks contraction
    b. Overfilled bladder
    c. Low lying placenta
    d. Placenta previa
A

a. Braxton Hicks contraction T transient myometrial contractions can simulate placenta previa; note thick myometrium; resolves with time. Can be seen at US from 8 weeks on (Obstetrics & the Newborn).b. Overfilled bladder T overly distended maternal bladder can mimic true placenta previa. BEST OPTION?
4. Antenatal scan at 20 weeks shows placenta over os. Technician asks you to check, and placenta is now clear of os. Which is most likely?
a. Braxton Hicks contraction T transient myometrial contractions can simulate placenta previa; note thick myometrium; resolves with time. Can be seen at US from 8 weeks on (Obstetrics & the Newborn).
b. Overfilled bladder T overly distended maternal bladder can mimic true placenta previa. BEST OPTION?
c. Low lying placenta F

d. Placenta previa F if true PP would stay over os (not transient).
PP is common in T2, with only a small minority persisting into late T3. Pics Callen p736

Full maternal bladder
• Approximates anterior and posterior uterine wallo Normally implanted placenta appears low
• Falsely elongates cervix
• Have patient void and repeat examFocal myometrial contraction
• Contraction can cause approximation of uterine wallso Similar to maternal full bladder
• May mimic placentao Can appear mass-like and echogenic
• Resolves with time
• TVUS can often differentiate cervix from contraction
o Slip of fluid seen at IO

How well did you know this?
1
Not at all
2
3
4
5
Perfectly
5
Q
  1. Fetal hydrops, normal stomach bubble and heterogenous echogenic mass in chest (also recalled as cystic-solid mass on prenatal US). Which is most likely
    a. CPAM
    b. Cystic hygroma
    c. Bronchogenic cyst
    d. CDH
    e. BPS
A

a. CCAM T echogenic +/- cystic mass, mediastinal shift, hydrops (caval compression), polyhydramnios (oesophageal compression). StatDx says hydrops in < 10%.
5. Fetal hydrops, normal stomach bubble and heterogenous echogenic mass in chest. Which is most likely
a. CPAM T echogenic +/- cystic mass, mediastinal shift, hydrops (caval compression), polyhydramnios (oesophageal compression). StatDx says hydrops in < 10%.
b. Cystic hygroma
c. Bronchogenic cyst

d. CDH *AJL - false - there is a normal stomch bubble so CDH is unlikely.
(Previous answer: less T Hydrops uncommon unless associated malformations present)

e. BPS less T hydrops may occur from cardiovascular compression(Added D&E)

How well did you know this?
1
Not at all
2
3
4
5
Perfectly
6
Q
  1. Postmenopausal woman (55 year old) with hirsutism. Ovarian mass. Most likely:
    a. Sertoli-leydig
    b. Leydig
    c. Thecoma
    d. Granulosa cell tumor
    e. Mucinous cystadenoma
A

a. Sertoli-leydig can produce testosterone & virilise
6. Postmenopausal woman (55 year old) with hirsutism. Ovarian mass. Most likely:
a. Sertoli-leydig can produce testosterone & virilise
b. Leydig – as above, but pure Leydig cell tumour (Hilus cell tumours) are rare
c. Thecoma – produce oestrogen; results in precocious puberty, endometrial hyperplasia/carcinoma
d. Granulosa cell tumor– as above
e. Mucinous cystadenoma – non-functional

How well did you know this?
1
Not at all
2
3
4
5
Perfectly
7
Q

Woman on IVF. U/S shows multiple cysts in left adnexa. U/S is routine.

a. normal stimulation.
b. hyperstimulation.
c. mucinous tumour
d. hydrosalpinx

A

• A = stimulation = normal to see multiple follicles/cysts while receiving stimulating hormones

Note:
• B = OHSS
signs:
o Bilateral ovarian enlargement (≥ 5cm, often > 10cm)
o Numerous large follicular cysts with dramatically increased local blood flow (some cysts may contain haemorrhage)
o Echogenic stromal tissue
o Ascites & effusions

How well did you know this?
1
Not at all
2
3
4
5
Perfectly
8
Q

MRI features of endocervical canal.

a. high T1.
b. central low T2 with peripheral high T2
c. parametrical extension is equivalent to 3b
d. cervical cancer is lower T2 than the rest of the normal cervix.

A

D = Kind-of T = high-signal intensity mass within low-signal intensity cervical stroma on T2WI; Roth – the tumour is intermediate in T2 signal – brighter than the inner fibromuscular stroma & darker than normal cervical mucosa

A = F on T1WI’s the whole uterus exhibits intermediate signal

B = F endocervical canal hyperintense, mid layer hypointense, outer layer intermediate T2 signal

C = F parametrial extension is FIGO stage II (but not to pelvic wall)

D = Kind-of T = high-signal intensity mass within low-signal intensity cervical stroma on T2WI; Roth – the tumour is intermediate in T2 signal – brighter than the inner fibromuscular stroma & darker than normal cervical mucosaEndocervical canal- mucosa : high- muscle : low- outer layer : intermediate

How well did you know this?
1
Not at all
2
3
4
5
Perfectly
9
Q
  1. Cervical cancer staging
    a. Stage I invades the parametrium but not the lower third of vagina
    b. Stage IIIB involves the bladder mucosa
    c. Stage IVA involves rectal mucosa
    d. Stage IIIA extends beyond the true pelvis
    e. Stage II involves the lower third of the vagina
A

c. Stage IVA involves rectal mucosa

1 a - microscopic
1 b - macroscopic
2 a - upper 2/3 vagina
2 b - parametrium
3 a - low 1/3 vagina
3 b - obstructive uropathy
4 a - bowel bladder invasion
4 b - met
How well did you know this?
1
Not at all
2
3
4
5
Perfectly
10
Q
  1. Incidence of ectopic pregnancy (not assisted reproduction):
    a. 1:50
    b. 1: 150
    c. 1: 500
    d. 1: 1500
A

a. 1:50

Dahnert: 2% of all pregnancies.

How well did you know this?
1
Not at all
2
3
4
5
Perfectly
11
Q
  1. A 60 year old female presents to the emergency department with RIF pain. Ultrasound demonstrated a hyperechoic rounded lesion in the right adnexa with cystic areas and no shadowing. This is most in keeping with:
    a. Ovarian torsion
    b. Ovarian metastasis
    c. Acute appendicitis
    d. Ovarian dermoid
    e. Haemorrhagic cyst
A

c. Acute appendicitis T possible if perforated with abscess (thick echogenic fluid +/- gas bubbles), can occur at any age
1. A 60 year old female presents to the emergency department with RIF pain. Ultrasound demonstrated a hyperechoic rounded lesion in the right adnexa with cystic areas and no shadowing. This is most in keeping with:
a. Ovarian torsion F usually occur in reproductive age & 1st three decades of life
b. Ovarian metastasis ?F ovary mets more common in premenopausal women due to vascularity of ovaries. However a possibility, typically from endometrium, breast, GIT or fallopian tube.
c. Acute appendicitis T possible if perforated with abscess (thick echogenic fluid +/- gas bubbles), can occur at any age
d. Ovarian dermoid F unlikely without any shadowing; typically stop growing at menopause & are most common in women < 45 years
e. Haemorrhagic cyst F wrong age group

How well did you know this?
1
Not at all
2
3
4
5
Perfectly
12
Q
  1. 36 female for first trimester screening ultrasound. Which does NOT affect risk of aneuploidy?

a. Maternal age
b. Nuchal translucency
c. Ossification of nasal bones (or “unossified nasal bones”)
d. Length of middle phalanx of the little finger
e. Twin pregnancy

A

d. Length of middle phalanx of the little finger ?F this is a sign in the 2nd trimester = clinodactyly (hypoplasia of middle phalanx of 5th finger). Nomograms for 17-26 weeks. Seen in 2-4% of normal hands and 60% of fetuses with T21.
e. Twin pregnancy F In dizygotic pregnancies, the maternal age-related risk for chromosomal abnormalities for each twin is the same as in singleton pregnancies and therefore the chance that at least one fetus is affected by a chromosomal defect is twice as high as in singleton pregnancies. In monozygotic twins, the risk for chromosomal abnormalities is the same as in singleton pregnancies and in the vast majority of cases both fetuses are affected. Correct response is referring to aneuploidy risk for individual twin, but overall for the pregnancy the risk of aneuploidy (of either twin) is doubled)
2. 36 female for first trimester screening ultrasound. Which does NOT affect risk of aneuploidy? (source FMF UK / StatDx)
a. Maternal age T
b. Nuchal translucency T
c. Ossification of nasal bones (or “unossified nasal bones”) T nasal bones not visible in T1 in 60-70% of fetuses with T21 & 2% of chromosomally-normal fetuses. At 15-20 weeks the nasal bones should both be > 2.5mm in length.
d. Length of middle phalanx of the little finger ?F this is a sign in the 2nd trimester = clinodactyly (hypoplasia of middle phalanx of 5th finger). Nomograms for 17-26 weeks. Seen in 2-4% of normal hands and 60% of fetuses with T21.
e. Twin pregnancy F In dizygotic pregnancies, the maternal age-related risk for chromosomal abnormalities for each twin is the same as in singleton pregnancies and therefore the chance that at least one fetus is affected by a chromosomal defect is twice as high as in singleton pregnancies. In monozygotic twins, the risk for chromosomal abnormalities is the same as in singleton pregnancies and in the vast majority of cases both fetuses are affected. Correct response is referring to aneuploidy risk for individual twin, but overall for the pregnancy the risk of aneuploidy (of either twin) is doubled)

How well did you know this?
1
Not at all
2
3
4
5
Perfectly
13
Q
  1. 28 female with repeated spontaneous abortions occurring in the second trimester. HSG demonstrates a Y shaped uterus with both Fallopian tubes present and patent. Which is MOST correct?

a. Arcuate uterus
b. Bicornuate uterus
c. Uterus didelphysis
d. Septate uterus
e. Unicornuate uterus

A

d. Septate uterus T there is significant overlap with the appearances of bicornuate uterus on HSG and need US or MRI to accurately delineate, thus given additional Hx, septate is favoured given Associated with recurrent spontaneous abortions (32-94%, 65% pooled spontaneous abortion rate).
3. 28 female with repeated spontaneous abortions occurring in the second trimester. HSG demonstrates a Y shaped uterus with both Fallopian tubes present and patent. Which is MOST correct?
a. Arcuate uterus F
b. Bicornuate uterus F Pregnancy outcomes have been reported to be close to those of the general population. However, some women do develop complications, such as pregnancy loss, preterm labor, or malpresentations. [UTD].
c. Uterus didelphysis F have 2 separate cervices & uterine cornua that are widely divergent. Spontaneous abortion rate 30%. Often the horns are more widely divergent c.f. septate uteri.
d. Septate uterus T there is significant overlap with the appearances of bicornuate uterus. Associated with recurrent spontaneous abortions (32-94%, 65% pooled spontaneous abortion rate).
e. Unicornuate uterus F

How well did you know this?
1
Not at all
2
3
4
5
Perfectly
14
Q
  1. 22 female approximately 8-10 weeks pregnant by dates presents with pelvic pain. Ultrasound confirms a live intrauterine gestation of 10 weeks gestation. A left adnexal 6 cm cyst is shown with peripheral vascularity and heterogenous internal echoes. Which is the MOST likely cause?

a. Corpus luteum
b. Corpus luteum with haemorrhage
c. Torted ovary
d. Heterotopic pregnancy
e. Dermoid cyst

A

b. Corpus luteum with haemorrhage T4.

22 female approximately 8-10 weeks pregnant by dates presents with pelvic pain. Ultrasound confirms a live intrauterine gestation of 10 weeks gestation. A left adnexal 6 cm cyst is shown with peripheral vascularity and heterogenous internal echoes. Which is the MOST likely cause?

a. Corpus luteum F
b. Corpus luteum with haemorrhage T
c. Torted ovary F increased risk during pregnancy, however atypical
d. Heterotopic pregnancy F possible , but risk 1:7,000 with spontaneous pregnancies, being much higher in IVF (affects 1-2% of IVF pregnancies)
e. Dermoid cyst F? possible. Look for echogenic Rokitansky nodule etc.US Req p309 – the most common adnexal mass, the CL cyst, is usually < 3cm (but can rarely reach 8cm), peak size at 7 weeks & resolves sonographically by mid 2nd trimester.US Req p366 – the appearance of a CL cyst, if complex & esp, with a thick rim, can overlap with adnexal ectopic pregnancy – key feature is can it be separated from the ovary (if not CL cyst, if can separate ectopic likely)

How well did you know this?
1
Not at all
2
3
4
5
Perfectly
15
Q
  1. 25 year old female has a second trimester ultrasound which demonstrates an echogenic mass within thorax on the left with a small systemic artery. Associated hydrops. (VIC – stomach normal place)

a. CCAM
b. Extralobar pulmonary sequestration
c. Congenital diaphragmatic hernia
d. Neuroblastoma
e. Rhabdomyosarcoma

A

b. Extralobar pulmonary sequestration T hydrops may occur from cardiovascular compression
6. 25 year old female has a second trimester ultrasound which demonstrates an echogenic mass within thorax on the left with a small systemic artery. Associated hydrops. (VIC – stomach normal place)
a. CCAM F no systemic supply; hydrops more common
b. Extralobar pulmonary sequestration T hydrops may occur from cardiovascular compression
c. Congenital diaphragmatic hernia F Hydrops uncommon unless associated malformations present
d. Neuroblastoma
e. Rhabdomyosarcoma

How well did you know this?
1
Not at all
2
3
4
5
Perfectly
16
Q
  1. Morphology scan at 20 weeks. Placenta is anterior and measures 1 cm from the cervix. The cervix is nodular and bulky. Which is the MOST likely explanation?

a. Braxton Hicks contraction.
b. Full bladder.
c. Low lying placenta.
d. Placenta praevia.
e. Lower uterine segment leiomyoma.

A

a. Braxton Hicks contraction. T can appear mass-like & echogenic
7. Morphology scan at 20 weeks. Placenta is anterior and measures 1 cm from the cervix. The cervix is nodular and bulky. Which is the MOST likely explanation?

a. Braxton Hicks contraction. T can appear mass-like & echogenic
b. Full bladder. F check again with bladder empty, but wouldn’t account for nodularity
c. Low lying placenta. T < 2cm from internal os
d. Placenta praevia. F not covering os
e. Lower uterine segment leiomyoma. T fibroids can affect cervix

How well did you know this?
1
Not at all
2
3
4
5
Perfectly
17
Q
  1. You go on a relief trip to the Phillipines after a massive earthquake. A 19 year old girl presents to the clinic with pelvic pain. She is afebrile. Ultrasound reveals bilateral thick walled cystic adnexal masses. Which is MOST likely?
    a. Bilateral ovarian cysts.
    b. Endometriosis.
    c. Tuberculous salpingitis.
    d. Polycystic ovarian syndrome.
    e. Hydrosalpinx.
A

c. Tuberculous salpingitis. T Bilateral complex cystic and solid adnexal masses ± calcification (TB TOAs)8.

You go on a relief trip to the Phillipines after a massive earthquake. A 19 year old girl presents to the clinic with pelvic pain. She is afebrile. Ultrasound reveals bilateral thick walled cystic adnexal masses. Which is MOST likely?

a. Bilateral ovarian cysts. F
b. Endometriosis. F usually diffuse low-level internal echoes
c. Tuberculous salpingitis. T Bilateral complex cystic and solid adnexal masses ± calcification (TB TOAs)
d. Polycystic ovarian syndrome. F
e. Hydrosalpinx. ?F usually tortuous/serpiginous, “beads on a string”

How well did you know this?
1
Not at all
2
3
4
5
Perfectly
18
Q
  1. During a routine 20 week morphology scan, a small mass is identified at the junction of the umbilical cord and anterior abdominal wall. The cord insert in the centre of the mass. Which is MOST likely?

a. AVM of the cord
b. Gastroschisis
c. Normal physiologic herniation
d. Omphalocoele
e. Pseudo-omphalocoele

A

d. Omphalocoele T cord inserts onto mass
9. During a rountine 20 week morphology scan, a small mass is identified at the junction of the umbilical cord and anterior abdominal wall. The cord insert in the centre of the mass. Which is MOST likely?
a. AVM of the cord F can’t find in Req, Callen or StatDx!
b. Gastroschisis F beside cord insertion
c. Normal physiologic herniation F must be < 12 weeks & < 7mm diameter
d. Omphalocoele T cord inserts onto mass
e. Pseudo-omphalocoele F = Deformation of the fetal abdomen by transducer pressure coupled with an oblique scan orientation

How well did you know this?
1
Not at all
2
3
4
5
Perfectly
19
Q
  1. 35 year old woman. Combined risk screening plus NT of 2.5 says risk 1:280 with background risk of 1:280. What do you do?

a. Talk to referrer
b. Report & get clerical staff to make sure referrer gets it
c. Borderline normal risk in report
d. Note normal in report

A

a. Talk to referrer TFMF UK say “screen-positive group cut-off of 1 in 300”
– this cut-off is also used in Queensland.

This will detect 80% of T21 fetuses & have a false positive rate of 5%. The estimated risk for trisomy 21 was 1 in 300 or more in 8% of the normal pregnancies, in 82% of those with trisomy 21 and in 78% with other chromosomal defects. For a screen-positive rate of 5%, the detection rate was 77% (95% confidence interval 72–82%).
⇨ Patient high-risk, need to discuss with patient & be offered invasive testing

.“… those preparing the report have obligations to report unusual, urgent or significant unexpected findings to the referrer in a timely manner. The timeliness is determined by the severity of the finding in the clinical context.” (RANZCR Standards of Practice for Diagnostic and Interventional Radiology Version 9.1, (2009) 33)

How well did you know this?
1
Not at all
2
3
4
5
Perfectly
20
Q
  1. 20 week US shows hydrops. Rounded homogenously echogenic mass in left thorax. Stomach bubble not identified. Most likely:
    i) CCAM
    ii) CDH
    iii) Myocardial issue
A

CDH or CCAMI would prefer CDH1)

20 week US shows hydrops. Rounded homogenously echogenic mass in left thorax. Stomach bubble not identified. Most likely:

i) CCAM T microcystic form uniformly echogenic, well defined, hydrops occurs not uncommonly. If large a potential cause of absent stomach bubble by compression of oesophagus.
ii) CDH ?T usually not uniformly echogenic (unless maybe contains only liver); absence of stomach bubble raises suspicion though; hydrops uncommon unless associated malformations present (Statdx)
iii) Myocardial issue F

Causes of absent stomach bubble ⇒ always do follow-up US
•	Physiologic (< 30min)
•	Oligohydramnios
•	Swallowing abnormality 
o	CNS defect
o	Neck/chest mass
•	Esophageal atresia
•	Congenital diaphragmatic hernia (CDH)
•	Situs abnormality

Causes of hydrops fetalis
• Immune hydrops (Rh-mediated)
• Non-immune hydrops:
o Cardiac (25-30%)
• Structural CHD
• Arrhythmia
• High-output failure (teratoma, AVM, anaemia)
o Chromosomal (15%)
• Trisomy 21/18/13
• Turner syndrome
o Infection (parvovirus B19, TORCH)
o Chest anomalies (CCAM → mass effect)
o Urinary tract obstruction (uncommon cause)
o ↓ oncotic pressure (nephrotic syndrome, hepatitis)
o TTTS
o α-thalassaemia (homozygous form)

How well did you know this?
1
Not at all
2
3
4
5
Perfectly
21
Q
  1. Echogenic bowel on 20 week scan. Most correct:
    i) Unlikely T21
    ii) As isolated finding suggests aneuploidy
    iii) Likely T13/18
    iv) CF
A

iv) CF T
2) Echogenic bowel on 20 week scan. Most correct:
i) Unlikely T21 F isolated echogenic bowel = 7x’s increased risk of T21 (FMF)
ii) As isolated finding suggests aneuploidy F soft marker – seen in 0.5-1% of normal US; more likely T21 if other markers seen
iii) Likely T13/18 F
iv) CF T

Echogenic bowel causes
•	Normal variant during 2nd trimester (transient inspissation; 0.5-1% of normal fetuses)
•	Genetic/syndromic causes
o	Cystic fibrosis 
o	Down syndrome (rare but has been reported)
•	Systemic causes
o	IUGR
o	CMV infection
o	Fetal hypoxia
o	Hydrops
•	Local factors
o	Intra-amniotic haemorrhage (ingested blood)
o	Early ascites
o	Meconium peritonitis
How well did you know this?
1
Not at all
2
3
4
5
Perfectly
22
Q
  1. 38yo female 11/40. NT scan shows 2.4mm nuchal thickness. With BHCG and PAPP-A gives risk of Aneuploidy of 1/281. Age related risk assessed at 1/281 as well. Advice for patient:
    i) NT risk same as age related risk – no further tests
    ii) High risk – advise CVS/amnio
    iii) Low risk - advise CVS/amnio
    iv) High risk - terminate
A

ii) High risk – advise CVS/amnio Patient high-risk, need to discuss with patient/O&G & be offered invasive testing

FMF UK say “screen-positive group cut-off of 1 in 300” – this cut-off is also used in Queensland. This will detect 80% of T21 fetuses & have a false positive rate of 5%. The estimated risk for trisomy 21 was 1 in 300 or more in 8% of the normal pregnancies, in 82% of those with trisomy 21 and in 78% with other chromosomal defects. For a screen-positive rate of 5%, the detection rate was 77% (95% confidence interval 72–82%).

How well did you know this?
1
Not at all
2
3
4
5
Perfectly
23
Q
  1. 25 yo obese female. No luck getting pregnant over 15months (probably can’t get a root). Findings on ultrasound (? Most likely or least likely ?):

a. Both ovaries demonstrate 20 follicles all less than 5mm
b. Collapsed ovarian cyst and fluid in POD
c. Simple (anechoic) 4cm cyst left ovary
d. Adnexal cyst
e. Cyst with low-level echoes measuring 4cm in diameter
f. POD cystic lesion

A

a. Both ovaries demonstrate 20 follicles all less than 5mm PCOS = 2003
* AJL - I have updated the question to reflect the more up to date criteria as outlined below (from radiopaedia)

The updated diagnostic criteria are based on a 2018 international consensus guideline: In patients >8 years post menarche, and using a high-frequency endovaginal probe:
- Follicle number per ovary (FNPO) ≥ 20,
and/or
- Ovarian volume ≥10 mL, ensuring no corpora lutea, cysts or dominant follicles are present

4) 25 yo obese female. No luck getting pregnant over 15months (Most likely) findings on ultrasound:

a. Both ovaries demonstrate 20 follicles all less than 5mm PCOS
(See above). (Previous answer discussed Rotterdam criteria which has been superseeded)

b. Collapsed ovarian cyst and fluid in POD F this would suggest ovulation
c. Simple (anechoic) 4cm cyst left ovary F would suggest persisting dominant follicle or benign neoplasm
d. Adnexal cyst ?T possible
e. Cyst with low-level echoes measuring 4cm in diameter T endometrioma possible, and could explain infertility
f. POD cystic lesion ?T possible

How well did you know this?
1
Not at all
2
3
4
5
Perfectly
24
Q
  1. Molar pregnancy (which is true?)

a. Never see fetal part in complete mole
b. Partial mole diploid
c. Association with bilateral theca lutein cysts
d. 30% invasive or malignant
e. Partial moles are at increased risk of malignant transformation.

A

c. Association with bilateral theca lutein cysts T – more common in complete mole – 25-60%
5) Molar pregnancy

a. Never see fetal part in complete mole F True if singleton pregnancy complete mole no fetal tissue present; False if coexist with a normal pregnancy (rare)
* LW think is also true, and would ignore the above dual pregnancy rarity.

b. Partial mole diploid F triploid (see below)
c. Association with bilateral theca lutein cysts T – more common in complete mole – 25-60%
d. 30% invasive or malignant F Pocket Radiologist OB says up to 20% become invasive or metastatic (choriocarcinoma)
e. Partial moles are at increased risk of malignant transformation. F (complete 20% risk of transformation into invasive mole or choriocarcinoma; although partial moles have an increased risk of persistent molar disease, they are not considered to have an increased risk for choriocarcinoma.)

How well did you know this?
1
Not at all
2
3
4
5
Perfectly
25
Q
  1. Pt presents for nuchal translucency scan. Bowel between abdo wall and cord insertion.

a. Normal bowel herniation
b. Omphalocele
c. Gastroschisis
d. Pseudo-omphalocele

A

b. Omphalocele T if scanned ≥ 12 weeks
6) Pt presents for nuchal translucency scan. Bowel between abdo wall and cord insertion.
a. Normal bowel herniation T if scanned < 12 weeks (11 weeks 5 days by FMF)
b. Omphalocele T if scanned ≥ 12 weeks
c. Gastroschisis F
d. Pseudo-omphalocele F technical due to probe or other pressure creating impression of omphalocele

Bowerman and associates – normal must be <7mm at any age. Any cord mass when CRL >44mm (11.1 weeks) is presumed abnormalNuchal scan performed 11+1 and 13+6.Thus the answer here is likely omphalocele.

How well did you know this?
1
Not at all
2
3
4
5
Perfectly
26
Q
  1. 20wk gestation. Placenta 1cm from internal os. Cervix 4cm long.
    a) Low lying placenta
    b) Extremely full bladder
    c) Placenta praevia
    d) Circumvellate placenta
A

a) Low lying placenta T
7) 20wk gestation. Placenta 1cm from internal os. Cervix 4cm long.
a) Low lying placenta T

b) Extremely full bladder ?T;
* LW: felt less likely as although can cause “abnormal placental location”, it can also cause apparent lengthening of the cervix, and is listed at 4cm - normal, so would assume this is why cervical length listed in question stem, to help exclude it from more correct option of A).

c) Placenta praevia F does not cover any part of os
d) Circumvellate placenta F placenta margin elevated along with membranes

Abnormal placental location
•	Technical/mimic
o	Full maternal bladder 
o	Uterine contractions (transient)
o	Placental abruption

• Placenta previa spectrum
o Low-lying placenta: ≤ 2cm from internal os, but does not cover or reach it
o Marginal placenta: covers part of cervix but not internal os
o Partial previa: partially covers internal os
o Complete previa: total coverage of internal os (almost never resolves)

How well did you know this?
1
Not at all
2
3
4
5
Perfectly
27
Q
  1. Pregnant woman with painless bleeding
    a) Placental abruption
    b) Placenta praevia
    c) Placenta percreta
    d) Placenta accrete
A

b) Placenta praevia T most commonly presents as painless bleeding in T
38) Pregnant woman with painless bleeding
a) Placental abruption F usually painful – StatDx says 80% present with vaginal bleeding +/- pain, while 20% present with pain & no bleeding
b) Placenta praevia T most commonly presents as painless bleeding in T3
c) Placenta percreta F classic presentation is uncontrollable hemorrhage in 3rd stage of labor
d) Placenta accrete F classic presentation is uncontrollable hemorrhage in 3rd stage of labor

How well did you know this?
1
Not at all
2
3
4
5
Perfectly
28
Q
  1. Obstetric measurements

a) 1st trimester CRL more accurate dating than biometry in T2

A

a) 1st trimester CRL more accurate dating than biometry in T2 T Although MFMs at Mater use the nuchal scan for dating (more accurate measurements, standardized).

How well did you know this?
1
Not at all
2
3
4
5
Perfectly
29
Q
  1. Woman undergoing infertility work up. US showed normal size uterus. HSG shows banana shaped uterus with a single fallopian tube.
    a) unicornuate uterus
    b) septate uterus
    c) bicornuate uterus
    d) didelphyis
A

a) unicornuate uterus ?T At USS, often difficult to detect, esp. if not suspected. Findings are a small, elliptical uterus deviated to one side at US; HSG findings fit though. High rate of obstetric complications.
11) Woman undergoing infertility work up. US showed normal size uterus. HSG shows banana shaped uterus with a single fallopian tube.
a) unicornuate uterus ?T At USS, often difficult to detect, esp. if not suspected. Findings are a small, elliptical uterus deviated to one side at US; HSG findings fit though. High rate of obstetric complications.
b) septate uterus F both cavities should fill at HSG, unless complete septum + vaginal septum
c) bicornuate uterus ? T – can be bicornuate bicollis, having cannulated single cervix; at US widely divergent, symmetric echogenic endometrial complexes
d) didelphys ? T – two distinct cervices, may cannulate only single cervix; however at US would see divergent uterine horns

How well did you know this?
1
Not at all
2
3
4
5
Perfectly
30
Q
  1. What favor mucinous rather than serous cystadenocarcinoma of ovary
    a) Another cyst on the contralateral side
    b) Stipple calcification
    c) Known BRCA1
    d) Multiple cysts >20
    e) Papillary projections and solid component
A

d) Multiple cysts >20 T mucinous tumours typically larger with more cysts containing viscous mucin12)

What favor mucinous rather than serous cystadenocarcinoma of ovary

a) Another cyst on the contralateral side F bilaterality uncommon for mucinous, 20-70% (frequent) for serous (two-thirds of serous cystadenocarcinomas are bilateral; 5% of mucinous cystadenocarcinoma bilateral)
b) Stipple calcification F calcification characteristic of serous tumour, not mucinous tumour
c) Known BRCA1 F serous more common
d) Multiple cysts >20 T mucinous tumours typically larger with more cysts containing viscous mucin
e) Papillary projections and solid component F papillary projections more common with serous tumours (StatDx)

How well did you know this?
1
Not at all
2
3
4
5
Perfectly
31
Q
  1. Which of the following is NOT associated with fetal ascites?
    a. Diaphragmatic hernia
    b. Parvovirus infection
    c. Urinary tract obstruction
    d. Multicystic dysplastic kidney
    e. Rhesus incompatibility
A

d. Multicystic dysplastic kidney
1. Which of the following is NOT associated with fetal ascites?
a. Diaphragmatic hernia T Hydrops uncommon unless associated malformations present
b. Parvovirus infection T the most common infectious cause of hydrops
c. Urinary tract obstruction T fetal ascites secondary to obstruction
d. Multicystic dysplastic kidney F
e. Rhesus incompatibility T immune hydropsHydrops = excess of total body fluid (2 or more abnormal fetal fluid collections: ascites; pleural effusion; pericardial effusion; skin oedema; polyhydramnios). Immune vs non-immune. 20% cases idiopathic. 40% also have detectable structural anomaly. Prognosis is poor if not treated. 75% survival for immune hydrops if treated with blood transfusion. Near 100% fatal if hydrops + fetal anomaly (excluding tachyarrhythmia).

How well did you know this?
1
Not at all
2
3
4
5
Perfectly
32
Q
  1. Which of the following WILL NOT reduce the fetal radiation dose of a pregnant woman who is undergoing a CTPA for suspected pulmonary embolism?

a. Decreased the mAV (?mAs)
b. Not doing the lateral scout
c. Increase scan pitch
d. Lead shielding to protect the abdomen
e. Stop scan field higher to avoid the upper abdomen

A
  • AJL - I would favour lead shielding to be the most useless of all the methods described below and therefore the answer to this question.
  • LW - agree as we no longer do this option here at CDHB given the near minimal effect of prtection / dose reduction offered.

All methods will reduce fetal dose, probably B would reduce fetal dose the least.
b. Not doing the lateral scout T utilize a PA projection where possible, and avoid directly scanning fetus

  1. Which of the following WILL NOT reduce the fetal radiation dose of a pregnant woman who is undergoing a CTPA for suspected pulmonary embolism?
    a. Decreased the mAs T++ reduced quantity of x-rays (also can ↓ kVp)
    b. Not doing the lateral scout T utilize a PA projection where possible, and avoid directly scanning fetus
    c. Increase scan pitch T increasing pitch reduces scan time & patient dose
    d. Lead shielding to protect the abdomen T+ can also use oral barium
    e. Stop scan field higher to avoid the upper abdomen T++ shorter scan length, stop before fetus is irradiated

All methods will reduce fetal dose, probably B would reduce fetal dose the least.

How well did you know this?
1
Not at all
2
3
4
5
Perfectly
33
Q
  1. 50yo woman presents to GP, unwell for several months. Reports a change in bowel and bladder habit, Clinical examination revealed abdominal distension. An US demonstrated bilateral complex pelvic masses and ascites. Which of the following is MOST LIKELY Dx?
    a. Serous cystadenocarcinoma
    b. Mucinous cystadenocarcinoma
    c. Germ cell tumour
    d. Endometrioma
    e. Ovarian fibroma
A

a. Serous cystadenocarcinoma

How well did you know this?
1
Not at all
2
3
4
5
Perfectly
34
Q
  1. A 48yo woman presents to GP with several months history of dysmenorrhoea and menorrhagia. On TV US, tender to transducer pressure, endometrium measured 4mm, myometrium was diffusely heterogeneous, and the serosal contour was smooth. Which is MOST LIKELY diagnosis?
    f. Adenomyosis
    g. Leiomyoma
    h. Endometrial carcinoma
    i. Endometrial polyp
    j. Endometritis
A

a. Adenomyosis T Ill-defined areas of myometrial heterogeneity with decreased echogenicity ± small myometrial cysts. 5th and 6th decade. Dysmenorrhea (30%), menorrhagia (50%), metrorrhagia (20%).

35
Q
  1. Which of the following regarding twin-twin transfusion syndrome is most correct?
    a. May occur in dizygotic twins with dichorionic placentas
    b. May occur in monozygotic twins with monoamniotic placentas
    c. May occur in monozygotic twins with dichorionic placentas
    d. May occur in dizygotic twins with monochorionic placentas
    e. May occur in 20% of twin pregnancies
A

b. May occur in monozygotic twins with monoamniotic placentas T (??? Watch the wording)

– TTTS can occur in MCMA twins, but is rarer than in MCDA twins & harder to diagnose

Which of the following regarding twin-twin transfusion syndrome is most correct?

a. May occur in dizygotic twins with dichorionic placentas F must be monochorionic
b. May occur in monozygotic twins with monoamniotic placentas T (??? Watch the wording) – TTTS can occur in MCMA twins, but is rarer than in MCDA twins & harder to diagnose
c. May occur in monozygotic twins with dichorionic placentas F must be monochorionic
d. May occur in dizygotic twins with monochorionic placentas F dizygotic always DCDA
e. May occur in 20% of twin pregnancies F occurs in 10-20% of monochorionic pregnancies (and only 30% of twin pregnancies are monozygotic, of which 70% are monochorionic)

36
Q
  1. 35yo female presented to ED with lower abdo pain, abdo distension, nausea and vomiting of a few hours duration. US showed intrauterine structures and ascites. Her bHCG level was markedly elevated. Which is the MOST LIKELY diagnosis?
    a. Single intrauterine pregnancy
    b. Endometrial carcinoma
    c. Multiple intrauterine pregnancy
    d. Uterine myoma
    e. Leiomyosarcoma
A

o Multiple intrauterine pregnancy T? increased incidence of hyperemesis gravidum

  1. 35yo female presented to ED with lower abdo pain, abdo distension, nausea and vomiting of a few hours duration. US showed intrauterine structures and ascites. Her bHCG level was markedly elevated. Which is the MOST LIKELY diagnosis?

o Single intrauterine pregnancy T? possible

o Endometrial carcinoma F wrong age

o Multiple intrauterine pregnancy T? increased incidence of hyperemesis gravidum

o Uterine myoma F usually asymptomatic

o Leiomyosarcoma F wrong presentation

37
Q
  1. Which of the following is LEAST LIKELY to be associated with increased incidence of urinary tract anomaly?
    a. Turners syndrome
    b. Single umbilical artery
    c. Hemihypertrophy
    d. Inperforate anus
    e. Medulloblastoma
A

e. Medulloblastoma

Which of the following is LEAST LIKELY to be associated with increased incidence of urinary tract anomaly?

a. Turners syndrome T renal ectopia and horseshoe kidney
b. Single umbilical artery T unilateral renal agenesis
c. Hemihypertrophy – T enlarged kidneys; renal medullary dysplasia; Wilms tumour
d. Inperforate anus T VACTERL
e. Medulloblastoma

38
Q
  1. A young Islander lady presents with pelvic pain and an IUD in situ. Her inflammatory markers and WCC count are elevated. Beta hCG negative. A pelvic ultrasound reveals a 4mm endometrium and small amount of free fluid in the Pouch of Douglas. The most likely pathology explaining these features is:
  2. Ruptured ectopic pregnancy
  3. Tubo-ovarian abscess
  4. Endometriosis
  5. Ruptured ovarian cyst
A
  1. Tubo-ovarian abscess - T - would account for raised inflam markers, and IUDs increase the risk of PID; would expect to see a complex adnexal mass if TOA, or other findings of gas in endometrium or hydropyosalpinx…
  2. A young Islander lady presents with pelvic pain and an IUD in situ. Her inflammatory markers and WCC count are elevated. Beta hCG negative. A pelvic ultrasound reveals a 4mm endometrium and small amount of free fluid in the Pouch of Douglas. The most likely pathology explaining these features is: (GC, TW)
  3. Ruptured ectopic pregnancy - F - beta-hCG negative.
  4. Tubo-ovarian abscess - T - would account for raised inflam markers, and IUDs increase the risk of PID; would expect to see a complex adnexal mass if TOA, or other findings of gas in endometrium or hydropyosalpinx…
  5. Endometriosis - F - wouldn’t account for raised inflam markers.
  6. Ruptured ovarian cyst - F - a 4mm endometrium is at upper range for menstrual phase (day 1-5).

Mirena IUD (one of the most commonly used nowadays) releases levonorgestrel (aka progestin, a synthetic progesterone) which thins the endometrium and reduces the frequency of ovulation. Some women develop ovarian cysts, but this wouldn’t account for raised inflam markers.

PID – ascending spread of microorganisms.
Risk factors: early age at sexual debut, multiple sexual parthers, Hx of STI. Chlamydia T and N. gonorrhoea (>50% have coinfection). Others: E.coli, HI. Peptostrep, TB.
Gonorrhoeal perihepataits = Fitz-Hugh Curtis. Complications = infertility 25% (8% after 1st episode, 20% after 2 episodes, 40% after 3 or more episodes PID). Ectopic pregnancy (6x increase). Chronic pelvic pain.

39
Q
  1. CCAM most likely
  2. at least one microcyst
  3. solid homogenous
  4. 5% congenital lung disease
  5. supplied by systemic vessel
  6. intact bronchial tree
A

5.intact bronchial tree – T? - intralobar mass of disorganised pulmonary tissue communicating with the bronchial tree. [Dahnert]
*LW: confusing regarding wording of this as definition of dysplastic within lesion that can connect to normal bronchial tree, but being dysplastic it is unlikely “intact” within the lesion.
Furthermore, previous combined answers in paeds / path MCQs quote although type 1 most common as macrocystic, these commonly have small/microcysts associated within them.
hopefully incorrect recall and was rather at least one MACROcyst.

CCAM types:
type I: most common: 70% of cases 3 large cysts one or more dominant cysts: 2-10 cm in size may be surrounded by smaller cysts

type II 15-20% of cases 3 cysts are <2 cm in diameter associated with other abnormalities renal agenesis or dysgenesis pulmonary sequestration congenital cardiac anomalies

type III~10% of cases microcysts: <5 mm in diameter typically involves an entire lobe has a poorer prognosis

type IV unlined cyst typically affects a single lobe indistinguishable from type I on imaging 11

type 0 very rare, lethal post natally acinar dysgenesis or dysplasia 11 represents global arrest of lung development 12

40
Q
  1. Hypoxic injury in premature neonate
  2. periventricular watershed
  3. external watershed
  4. peri rolandic infarction
A
  1. periventricular watershed T - vascular watershed (“border zone”) shifts from periventricular in premature to parasagittal in term infant at about 36wks. Also remember that before 26wks there is no gliosis, vs after 26 wks where develop PVL (vs porencephaly).
  2. Hypoxic injury in premature neonate (TW)
  3. periventricular watershed T - vascular watershed (“border zone”) shifts from periventricular in premature to parasagittal in term infant at about 36wks. Also remember that before 26wks there is no gliosis, vs after 26 wks where develop PVL (vs porencephaly).
  4. external watershed
  5. peri rolandic infarction - F - precentral, middle and postcentral gyri. Insult < 26 weeks – porencephalic cyst / >26 weeks get gliosis.Shift of blood supply from ventriculopetal to ventriculofubal at 36/40.
41
Q
  1. Sequestration
  2. RUL
  3. RLL
  4. LUL
  5. RML
  6. LLL
A

5.LLL - T - intralobar sequestration LLL > RLL 60%:40%. Extralobar sequestration ILS usually presents later than ELS. Patients with ILS usually present in late childhood or adolescence with recurrent pulmonary infections. ELS more apt to present in early life from resp distress / cyanosis / or assoc abnormalities. Both can be an incidental antenatal Dx too. 65% of ELS assoc with other anomalies. ILS not associated with other anomalies.

42
Q
  1. Molar pregnancy which is true?
  2. Partial mole is diploid.
  3. Fetal parts are seen in complete mole.
  4. Partial mole is associated with choriocarcinoma.
  5. Are associated with Bilateral theca lutein cysts.
A
  1. Are associated with bilateral theca lutein cysts - T - theca lutein cysts secondary to very high BhCG levels in up to 50% or cases (more commonly associated with complete moles). (UpToDate, Black book)
  2. Molar pregnancy which is true? (TW)
  3. Partial mole is diploid - F - most (90%) partial moles are triploid 69XXX, 69XXY, rarely 69XYY. Other 10% are a variety of other karyotypes - although the true existence of a nontriploid partial mole has been challenged.
  4. Fetal parts are seen in complete mole - F - fetal parts assoc with partial mole - and fetal or embryonic tissue may have a normal diploid or a triploid karyotype. No fetal tissue with complete mole, however either complete or partial mole can occur with twin pregnancy complicated by GTD (ie 2x moles, or mole + viable fetus).
  5. Partial mole is associated with choriocarcinoma - F - gestational trophoblastic neoplasia (includes choriocarcinoma, placental site tumor, and epitheloid trophoblastic tumor) usually occurs after a complete mole rather than a partial mole. Choriocarcinoma only rarely occurs after a partial mole.
    * AJL - Partial mole is not associated with choriocarcinoma (Robbins)

4.Are associated with bilateral theca lutein cysts - T - theca lutein cysts secondary to very high BhCG levels in up to 50% or cases (more commonly associated with complete moles). (UpToDate, Black book)

43
Q

6.Which of the following are true?

  1. Stuck fetus in twin-twin transfusion is the recipient fetus.
  2. On US the presence of two sacs each with a live fetus at 8 weeks means monochorionic diamniotic twins.
  3. The risk of pregnancy loss for monozygotic and dizygotic twins is the same.
  4. On US , if placental tissue is seen on a separating membrane between the fetuses the twinning is dichorionic.
  5. Twin-twin transfusion only occurs in monochorionic monamniotic twins.
A
  1. On US , if placental tissue is seen on a separating membrane between the fetuses the twinning is dichorionic - T - this constitutes the lambda or twin peak sign. Intertwin membrane is thick. Echogenic chorionic tissue extends into the membrane between the twins giving rise to the twin-peak or lambda sign.
  2. Which of the following are true? (TW)
  3. Stuck fetus in twin-twin transfusion is the recipient fetus - F - “stuck fetus” refers to the donor twin which becomes anemic, hypovolaemic and looses amniotic fluid. Close apposition of the inter-twin membrane fixes the donor fetus in a position.
  4. On US the presence of two sacs each with a live fetus at 8 weeks means monochorionic diamniotic twins - F - could be DCDA twins. Determination of MCMA, MCDA, DCDA depends on: heterozygous / homozygous; and if homozygous the division (first 3 days - DCDA, between 4th - 8th day MCDA, after 13th day - congoined twins). Note that should be able to determine MCDA vs DCDA on US at this stage however - twin-peak or lambda sign for DC, and T-sign for MC.
  5. The risk of pregnancy loss for monozygotic and dizygotic twins is the same - F - MC twins have 3-5x higher incidence (vs DC twins) of perinatal morbidity and mortality.
  6. On US , if placental tissue is seen on a separating membrane between the fetuses the twinning is dichorionic - T - this constitutes the lambda or twin peak sign. Intertwin membrane is thick. Echogenic chorionic tissue extends into the membrane between the twins giving rise to the twin-peak or lambda sign.
  7. Twin-twin transfusion only occurs in monochorionic monamniotic twins - F - occurs in 9-15% of monochorionic anteripregnancies including MCMA and MCDA. MC twinning with artery-to-vein anastomoses in placenta allowing donor twin to partly perfuse the recipient (Black book). Although MZ twins can have DC placentation, such twins are not at risk for TTTS.
44
Q
  1. Regarding polycystic ovarian syndrome?
  2. Normal size excludes diagnosis.
  3. Cysts tend to be peripheral.
  4. Cysts are predominantly 15-20mm in diameter.
  5. ovarian volumes tend to be <10cm3.
A
  1. Cysts tend to be peripheral - T - although there is a tendency to be peripheral, this, along with echogenic central stroma is no longer considered important. As long as ther are 12 or more follicles in each ovary. “string-of-pearls” appearance.
  2. Regarding polycystic ovarian syndrome? (TW)
  3. Normal size excludes diagnosis - F - clinical and biochemical criteria can still Dx. Note also that Rotterdam criteria ‘enlarged ovaries’ is >10mL (or >14mL as per black book). Not that normal ovarian volumes can be considered normal upt o 15mL and even up to 20mL in young nulliparous and older parous women (Ultrasound, the requisites 2nd ed).
  4. Cysts tend to be peripheral - T - although there is a tendency to be peripheral, this, along with echogenic central stroma is no longer considered important. As long as ther are 12 or more follicles in each ovary. “string-of-pearls” appearance.
  5. Cysts are predominantly 15-20mm in diameter - F - 2-9mm diameter
  6. ovarian volumes tend to be <10cm3 - F - >10mL. criteria- > 12 follicles / > 10 ml / ovary
45
Q

8.In regards to Sacrococcygeal teratomas?

  1. MRI is essential for diagnosis.
  2. 60% with calcification will benign.
  3. Anterior sacral meningocele is common.
  4. Follow post resection is with serial Beta HCG levels.
A
  1. 60% with calcification will benign.- T - calcifications are present in at least 60% of SCTs and occur more frequently in benign lesions. As about 75% of all SCTs are benign, presumably this will represent at least 60% of all, esp if calcification is more common in the benign group.
  2. In regards to Sacrococcygeal teratomas? (TW)
  3. MRI is essential for diagnosis - F - can be Dx with US
  4. 60% with calcification will benign.- T - calcifications are present in at least 60% of SCTs and occur more frequently in benign lesions. As about 75% of all SCTs are benign, presumably this will represent at least 60% of all, esp if calcification is more common in the benign group.
  5. Anterior sacral meningocele is common - F - SCTs: predominantly external lesion and minimal presacral component 47%; predominantly ext tumro with significant presacral component 35%; predominantly sacral component and external extension 8%; presacral tumor with no external component 10%. Anterior sacral meningocele - It represents 2.8% of all presacral tumors in females. Only case reports of prenatal diagnosis (therefore, uncommon)

4.Follow post resection is with serial Beta HCG levels - ?F - all patients with SGT should have an initial assay of serum tumor markers, and be monitored with serial tumor markers following resection of apparently benign teratomas.
Dahnert says SCT has elevated AFP with mixed malignant teratoma and endodermal sinus tumor. UpToDate says both AFP and BhCG.
Sacrococcygeal teratomas are the most common GCT of childhood, accounting for 40% of all GCTs and up to 78% of all extragonadal GCTs. Thought to arise from the primitive knowt or Hensen’s node (aggregation of totipotential cells that are the primary organizers of embryonic development).

46
Q

9.Regarding Pulmonary sequestration which is true?

  1. The intralobar form most frequently presents in neonate.
  2. Intralobar have arterial supply from pulmonary artery.
  3. Intralobar form drains to pulmonary veins.
  4. Extralobar occur most commonly on the right.
  5. Extralobar characterized by presence of air bronchograms.
A
  1. Intralobar form drains to pulmonary veins - T - ILS drains to LV, but can have abnormal drainage to IVC, azygous, or RA. ELS receives arterial supply from aberrant vessel arising from thoracic aorta, and has anomalous venous drainage to RA, vena cava, or azygous. 95% drain to pulmonary vein, others drain to azygos etc.
  2. Regarding Pulmonary sequestration which is true? (TW)
  3. The intralobar form most frequently presents in neonate - F - ILS usually presents later than ELS. Patients with ILS usually present in late childhood or adolescence with recurrent pulmonary infections. ELS more apt to present in early life from resp distress / cyanosis / or assoc abnormalities. Both can be an incidental antenatal Dx too.
  4. Intralobar have arterial supply from pulmonary artery - F - systemic arterial supply, usually lower thoracic or upper abdo aorta. Single or multiple arterial trunks. Venous drainage usual normal to left atrium, but can be abnormal and connect to IVC, azygous, or RA.
  5. Intralobar form drains to pulmonary veins - T - ILS drains to LV, but can have abnormal drainage to IVC, azygous, or RA. ELS receives arterial supply from aberrant vessel arising from thoracic aorta, and has anomalous venous drainage to RA, vena cava, or azygous. 95% drain to pulmonary vein, others drain to azygos etc.
  6. Extralobar occur most commonly on the right - F - ELS is almost always LLL (ILS LLL > RLL 60%:40%).

5.Extralobar characterized by presence of air bronchograms - F - lacks a bronchial connection to normal proximal airway. May connect to GIT or rarely, to intrpulmonary structures.
Infectious complications are uncommon in ELS.

o Intralobar sequestration – no pleural covering (contained within the substance of the lung)
• May be an acquired anomaly resulting from recurrent infections that produce aberrant arterial vessels arising from the aorta (controversial – StatDx says “vast majority” are acquired)
• More common form
• 50% diagnosed before 20 yrs, diagnosis rarely made after 20 yrs – may manifest as incidental fnding on CXR or with recurrent pneumonias
• Not assoc/ w/ other abnormalities
• Site: posterobasal segment of LLL is most common site➢ LLL > RLL➢ Posterobasal > mediobasal segment
• Vasculature:
➢ Arterial supply usually from aorta, but may receive systemic supply from diaphragm, chest wall or abdomen
➢ Venous drainage into inferior pulmonary vein, creating a left-to-left shunt (95% have pulmonary venous drainage)
o Extralobar sequestration – contained within its own pleural covering
• Congenital abnormality caused by failure of obliteration of one of the systemic arterial connections to the base of the developing fetal lung
• Manifests in neonates/infants, boys > girls
• Assoc/ w/ other congenital abnormalities in 65%
• Site: ➢ LLL > RLL➢ Typically occurs b/w LL & diaphragm – however may occur within diaphragm, mediastinum or pericardium, or rarely be subdiaphragmatic. Occasionally there may be a communication with the stomach or oesophagus.
• Vasculature:
➢ Arterial supply from descending thoracic aorta (subdiaphragmatic in 20%)➢ Venous drainage into systemic veins, typically azygos system

47
Q
  1. In multicystic dysplastic kidneys?
  2. There is increased risk of malignancy.
  3. In the pelvoinfundibular form , the cysts communicate.
  4. The ureter is atretic.
  5. Bilateral involvement is common.
  6. Associated with hepatic cysts.
A
  1. The ureter is atretic - T - more common pelvoinfuncibular form - there is atresia of the ureter or renal pelvis. In hydronephrotic form there is an atretic segment of ureter
  2. In multicystic dysplastic kidneys? (TW)
  3. There is increased risk of malignancy - F - although come clinicians have recommended resection to eliminate risk of malignancy, esp Wilms tumor, there appears to be no increased risk for Wilms tumor in MCDK (UpToDate) - also CME 02.113.
  4. In the pelvoinfundibular form , the cysts communicate - F - pelvoinfundibular MCDK (more common type) - results from atresia of ureter or renal pelvis, cysts are remnants of dilated calyces. Classic imaging - fetal flank mass with multiple cysts of varying sizes that do not connect. Hydronephrotic type may have a dominant cyst in the pelvis (CME 02.113)
  5. The ureter is atretic - T - more common pelvoinfuncibular form - there is atresia of the ureter or renal pelvis. In hydronephrotic form there is an atretic segment of ureter.
  6. Bilateral involvement is common - F - bilateral involvement is fatal. 40% of contralateral kidneys are abnormal (esp. contralateral PUJ obstruction; 20% contralateral MCDK, 10% contralateral renal agenesis, 10% contralateral hydronephrosis). 5.Associated with hepatic cysts - F
48
Q
  1. On four chamber echo view which is not seen?
  2. TOF
  3. VSD.
  4. ASD.
  5. Coarctation.
A
  1. TOF – T (SK) Pocket/StatDx – 4C view normal in > 95% of prenatal cases of TOF – outflow tract assessment is key to making diagnosis
  2. On four chamber (fetal?) echo view which is not seen? (TW) Callen p514-
  3. TOF – T (SK) Pocket/StatDx – 4C view normal in > 95% of prenatal cases of TOF – outflow tract assessment is key to making diagnosis
  4. VSD. – F Pocket – 30% missed on 4C view. RG 2002 – a large VSD is easily diagnosed on the 4C view alone
  5. ASD - ?F - this would be false if postnatal cardiac US. Foramen ovale is patent in fetus. A defect in the atrial septum is seldom mentioned in prenatal studies, because it can rarely be Dx antenatally. However, can Dx if large ASD.
  6. Coarctation. - ?T/F - although I guess the best view would be aortic arch view, findings on 4 chamber view may suggest coarctation. When the LV is very small, hypoplastic left heart is the most likely lesion. When the LV is only slightly small, coarctation of the aorta is the main lesion to consider (DDx TAPVR, IUGR, and forms of pulm atresia).
49
Q
  1. Down syndrome, false
  2. Persistent metopic suture
  3. Large acetabular angles
  4. Flared iliac wings
  5. Clinodactyly
A
  1. Large acetabular angles -Decreased acetabular angle secondary to flattened acetabular roof
  2. Downs, false (TW)
  3. Persistent metopic suture - T - 40-79% after age 10.
  4. Large acetabular angles -Decreased acetabular angle secondary to flattened acetabular roof
  5. Flared iliac wings - T - rotation of iliac wings toward coronal plane at SI joints.
  6. Clinodactyly - T - 50% = hypoplastic middle phalanx of 5th finger, distal finger curves inwards; widened space between first 2 digits of hands and feet (sandal gap foot)
50
Q
  1. Prune belly, false
  2. Abdo wall defect
  3. Cryptorchid
  4. Pulmonary hypoplasia
  5. Stricture posterior urethra
  6. Reflux
A

*LW:
See path / paeds MCQ: preferred option is stricture of posterior urethra which is FALSE.
Abdo wall defect is incorrect wording but more correct than posterior urethra stricture.

  1. Prune belly, false (TW/SK)
  2. Abdo wall defect – F? (SK) wrinkled, flaccid hypotonic abdominal wall with bulging flanks, but no focal defect/gap in the wall. There is absence or hypoplasia of the rectus muscles (is this a defect?).
  3. Cryptorchid - T - in males
  4. Pulmonary hypoplasia - T - reduced urine production - get oligohydramnios - get mechanical compression of fetus and inhibition of thoracic expansion. Without amniotic fluid entering airways, lungs remain hypoplastic.
  5. Stricture posterior urethra – T (TW) - hallmark of PBS in males is markedly hypoplastic or dysplastic prostate, which leads to dilatation of the prostatic urethra. In occasional patients, a valve-like mechanism develops in the membranous part of urethre and causes urinary obstuction. Dahnert – urethral obstruction in 20%.
  6. Reflux - T - ureteral peristalsis is ineffective or lacking. Get varying degrees renal insufficiency due to reflux and obstruction (if reflux referred to oesophageal - not sure if T or F. This answer was previously remembered as correct)

Prune-belly (Eagle-Barrett) syndrome - congenital disorder defined by a characteristic clinical triad: a) abdominal muscle deficiency, b) severe urinary tract abnormalities, c) bilateral cryptorchidism in males. 1: 40,000 – 50,000 births. Much more common in males (plus females can’t really have cryptorchidism - pseudoprune belly). Suggested underlying defect is abnormal mesoderm development - in particular a primary defect in the intermediate and lateral plate mesoderm. GUT - unilateral or bilateral abnormalities of ureter, prostate, bladder and testicles. Ureters dilated, elongated (replacement of smooth muscle with fibrous tissue). RG Sep 2002:There appear to be two groups of patients. The first group has an obstructing lesion of the urethra (urethral atresia or posterior urethral valves) that leads to death soon after birth. The second group presents with a functional abnormality of bladder emptying but no urethral obstruction and usually survives the neonatal period, developing chronic urinary tract disease. The first group is less numerous than the second group.

51
Q
  1. Cleft palate, which is false:
  2. 1:800
  3. Midline sagittal image is best
  4. Can be missed easily on USS if only finding
  5. ?Hydramnios
  6. Type 4 CL/P is most common type assoc with chromosomal abnormalities
A
  1. Midline sagittal image is best - F - angled coronal facial view through lip / nose - “snout view” - looking for linear or large defect of upper lip. Defect in palate seen best on axial/transverse view. Acutally have a use for 3DUS here too.
  2. Cleft palate, which is false: (TW) Pocket p65, US Req p387
  3. 1:800 - T – cleft lip with or without cleft palate: 1:600 Asian and native Americans, 1:1000 caucasian, 1:2500 african-american. Isolated cleft palate only occurs in 1:2500 newborns and does not display variation by ethnicity.
  4. Midline sagittal image is best - F - angled coronal facial view through lip / nose - “snout view” - looking for linear or large defect of upper lip. Defect in palate seen best on axial/transverse view. Acutally have a use for 3DUS here too.
  5. Can be missed easily on USS if only finding - T - cleft palate with an intact lip is the most difficult orofacial malformation to diagnose prenatally.
  6. Hydramnios - T - polyhydramnios when severe - intereferes with swallowing.
  7. Type 4 CL/P is most common type assoc with chromosomal abnormalities - T - rare for type 1, 20% for type 2, 30% for type 3, and 52% for type 4. Consider amniocentesis for types 2-4.

Type 1 - linear defect upper lip +/- nasal deformity.

Type 2 - defect in palate with nasal deformity.

Type 3 - very abnormal bilateral lip defect, always nasal deformtiy.

Type 4 - midline gap involving lip and palate.

Cleft lip with cleft palate comprises about 50%. Isolated cleft lip 25%. Isolated cleft palate 25%. Cleft lip with, or without cleft palate is more common than cleft palate and varies by ethnicity.

Embryology: CL - failure of normal fusion of 3 lip segments (2 lateral, 1 middle).
CL/P - CL plus failure of normal fusion of 3 primary palate segments. CP alone - failure of normal fusion of 2 secondary palate segments. Amniocentesis for karyotype should be offered to women wiht US findings of fetal orofacial clerts and assoc anomalies due to high rate of chromosomal defects. Orofacial clefts in the absence of other assoc congenital abnormalities are unlikely to be due to chromosomal abnormalities; however, in the difficulty in prenatal sonographic Dx of some assoc malformations supports chromosomal evaluation in all fetuses with prenatally Dx facial clefts. (UpToDate, Blackbook) Added option 5 - previously said “not available” type I: isolated cleft lip alone type II: unilateral cleft lip and palate type III: bilateral cleft lip and palatetype IV: midline/median cleft lip and palate type V: facial clefts associated with the amniotic band syndrome or the limb-body-wall complexIn radiology notes said only a/w trisomy 18 and 13

52
Q
  1. Regarding Turner’s Syndrome, which is not an association:
  2. Cervical spine abnormalities
  3. Horseshoe kidney
  4. Increased risk of adenocarcinoma of the bowel
  5. Early fusion of physes
  6. Thyromegaly
A
  1. Early fusion of physes - F - normal skeletal maturation with growth arrest at skeletal age of 15y. Delayed fusion of epiphyses > age 20y.
  2. Regarding Turner’s Syndrome, which is not an association: (TW)
  3. Cervical spine abnormalities T – hypoplasia of odontoid & C1
  4. Horseshoe kidney - T - approx 30-50% have a renal malformation, most commonly horseshoe kidney, followed by abnormal vascular supply.
  5. Increased risk of adenocarcinoma of the bowel – T? - (Nelsons Pediatrics) : In analysis of cohort of 597 Turners women (Danish Cytogenetic Register) there was relative risk of 6.9 for colon cancer. (UpToDate) : patients are at increased risk for gonadoblastoma. May also be at risk for other tumors - overall risk of cancer not increased, but sit-specific risks were increased for meningioma, childhood brain tumors, bladder, and uterine cancer (but not breast) - didn’t make mention of bowel.
  6. Early fusion of physes - F - normal skeletal maturation with growth arrest at skeletal age of 15y. Delayed fusion of epiphyses > age 20y.
  7. Thyromegaly - T - (Dahnert - thyromegaly). Often see hypothyroidism, positive thyroid autoantiboides.
53
Q
  1. Normal for dates on clinical assessment, choose the most correct
  2. Spina bifida
  3. Maternal DM
  4. Bilateral MCDK
  5. Duodenal atresia
  6. Anencephaly
A
  1. Spina bifida - T - most correct (unless is part of a more complex abnormality)
  2. Normal for dates on clinical assessment, choose the most correct (TW)
  3. Spina bifida - T - most correct (unless is part of a more complex abnormality)
  4. Maternal DM - F - macrosomic due to diabetes (gestational and pre-existing diabetes) - ie est weight > 90th centile, birth weight >4500g.
  5. Bilateral MCDK - F – Usually fatal. No renal production of fluid (major contributer) so oligohydramnios & uterus therefore small for dates.
  6. Duodenal atresia - F - Polyhydramnios
  7. Anencephaly - F - Polyhydramnios common due to impaired swallowing.

Amniotic fluid regulation reflects balance between production and removal. Major AF production: Fetal kidneys - 800-1200 cc/d near term; Fetal lungs - 170cc/d near term. Major AF removal: Fetal swallowing/intestines - 500-1000 cc/d near term; Fetal lungs; 170cc/d near term. Other major AF transfer pathways: Placenta, membranes, fetal skin, umbilical cord.

54
Q
  1. IUGR, choose the most correct
  2. Spina bifida
  3. Maternal DM
  4. MCDK
  5. Duodenal atresia
  6. Anencephaly
A

5.Anencephaly - T - best answer - no head.

IUGR, choose the most correct (TW)

  1. Spina bifida - F
  2. Maternal DM - F - big baby
  3. MCDK - F - unilateral MCDK - normal size
  4. Duodenal atresia - F - could argue may have smaller AC (less fluid)
  5. Anencephaly - T - best answer - no head. Doubled up these last 2 MCQs - as initial MCQ was ambiguous - didn’t clearly specify if talking about clinical small for dates, or US small for dates. Very different answers.
55
Q
  1. Which is not assoc with foetal ascites
  2. Diaphragmatic hernia
  3. Parvovirus
  4. Urinary tract obstruction
  5. Rhesus incompatability
  6. Multicystic dysplastic kidney
A
  1. Multicystic dysplastic kidney - F - unilateral MCDK often incidentally discovered during prenatal US. Note that 40% of MCDK assoc with contrlateral kidney problem, in
  2. Which is not assoc with foetal ascites (TW)
  3. Diaphragmatic hernia - T (ie assoc with) - thoracic lesions causing hydrops: CCAM, CDH, sequestration, chylothorax, lymphangiectasis, neoplasms, bronchogenic cyst. These increase intrathoracic pressure and can obstruct venous return to the heart - peripheral venous congestion, or can obstruct lymphatic duct, resulting in lymphedema.
  4. Parvovirus - T - parvovirus B19 is the most common infectious agent assoc with hydrops. TORCH pathogens (CMV most common of these) can also cause non-immune hydrops.
  5. Urinary tract obstruction - T - GUT malformations represent a very small proportion of non-immune hydrops. Disorders such as PUVs leading to prune belly sundrome may cause intraabdominal obstruction of venous return.
  6. Rhesus incompatability - T - immune hydrops.
  7. Multicystic dysplastic kidney - F - unilateral MCDK often incidentally discovered during prenatal US. Note that 40% of MCDK assoc with contrlateral kidney problem, in which case may result in problems (ie contralateral MCDK, PUJ, renal agenesis).

hydrops = excess of total body fluid (2 or more abnormal fetal fluid collections: ascites; pleural effusion; pericardial effusion; skin oedema; polyhydramnios). Immune vs non-immune. 20% cases idiopathic. 40% also have detectable structural anomaly. Prognosis is poor if not treated. 75% survival for immune hydrops if treated with blood transfusion. Near 100% fatal if hydrops + fetal anomaly (excluding tachyarrhythmia).

56
Q

19.CCAM which is false

  1. Surgery is usually recommended for asymptomatic patients iwth Dx of CCAM made on prenatal US
  2. Can cause fetal hydrops
  3. Commonly associated with other anomalies
  4. Associated with mediastinal shift
A

3.Commonly associated with other anomalies - F - 25% have associated structural anomalies, and fetal karyotype is indicated in these cases, esp type II. (UTD 2011 SK)

**LJS - management for smaller asymptomatic lesions is controversial. Often watch and wait rather than excision. I think 1. is more incorrect than 3.

  1. CCAM which is false (TW)
  2. Surgery is usually recommended for asymptomatic patients with Dx of CCAM made on prenatal US - T - UpToDate (Jan 2009) - early intervention is recommended to prevent the potential complications of recurrent infections and/or malignancy. In symptomatic patients immediate surgical intervention may be required.
  3. Can cause fetal hydrops - T - large CCAMs and shift of the mediastinum may lead to obstruction of the IVC and cardiac compression, resulting in development of hydrops from increased central venous pressure.
  4. Commonly associated with other anomalies - F - 25% have associated structural anomalies, and fetal karyotype is indicated in these cases, esp type II. (UTD 2011 SK)
  5. Associated with mediastinal shift - T - contralateral mediastinal shift in 90%

*LW:
annoyingly with regards to defintions, “common” is associated with 10% frequency, while “rare” 0.1%, I dont think this distracts from the above answer tho… just more useless info to confuse….

57
Q

21.Regarding the fetal gastrointestinal tract, which is the most correct?

  1. Rectal atresia can be associated with polyhydramnios
  2. Visualized stomach excludes esophageal atresia
  3. Tubular and cystic enteric duplications cysts have a similar incidence
  4. Omphalocele is associated with T21 and T18, but not T13
A
  1. Rectal atresia can be associated with polyhydramnios - T - although intestinal obstruction results in decreased resorption of amniotic fluid and development of polyhydramnios it tends to be with proximal bowel obtruction, rather than distal (colonic atresia may have polyhydramnios) UpToDate. Anal atresia is NOT usually associated with polyhydramnios, since an adequate length fo bowel is available for absorption of swallowed amniotic fluid. However, if part of VACTERL - polyhydramnios can occur (US in O&G, E Merz).
    * LW: hoping the correct option was not recalled, as although agree with two step process with VACTERAL, rectal atrsesia itself should not cause poly… so on principle I would have this as false, although depending on what unrecalled option was…
  2. Regarding the fetal gastrointestinal tract, which is the most correct? (TW)
  3. Rectal atresia can be associated with polyhydramnios - T - although intestinal obstruction results in decreased resorption of amniotic fluid and development of polyhydramnios it tends to be with proximal bowel obtruction, rather than distal (colonic atresia may have polyhydramnios) UpToDate. Anal atresia is NOT usually associated with polyhydramnios, since an adequate length fo bowel is available for absorption of swallowed amniotic fluid. However, if part of VACTERL - polyhydramnios can occur (US in O&G, E Merz).
  4. Visualized stomach excludes esophageal atresia - F - proximal oesophageal atresia with distal tracheoesophageal fistula is most common, and stomach may be present but small.
  5. Tubular and cystic enteric duplication cysts have a similar incidence - F - 80% cystic, 20% tubular
  6. Omphalocele is associated with T21 and T18, but not T13 - F - assoc with all.
58
Q
  1. Fetal PUJ, which is false:
  2. Bilateral hydroureter leads to a bad outcome
  3. Bilateral agenesis results in oligohydramnios
  4. PUJ is bilateral in 70%
  5. Hydrops leads to a bad outcome
A
  1. PUJ is bilateral in 70% - F - bilateral in 30% (Pocket)
  2. Fetal PUJ, which is false: (TW)
  3. Bilateral hydroureter leads to a bad outcome - T - not sure how this realtes to PUJ. If bilateral hydroureter - indicates a distal obstruction (eg PUVs), abnormal musculature / development (Prune Belly, congenital megaureter), reflux.
  4. Bilateral agenesis results in oligohydramnios - T - ?bilateral agenesis??. Fetal kidneys contribute 800-1200 mL/d near term. Without kidney output - develop oligohydramnios. Note however that with PUJ specifically, oligohydramnios is rare and polyhydramnios may be paradoxically present (obstruction may impair kidney’s ability to concentrate urine with subsequent high urine output)
  5. PUJ is bilateral in 70% - F - bilateral in 30% (Pocket)
  6. Hydrops leads to a bad outcome - T - hydrops = excess of total body fluid (2 or more abnormal fetal fluid collections: ascites; pleural effusion; pericardial effusion; skin oedema; polyhydramnios). Immune vs non-immune. 20% cases idiopathic. 40% also have detectable structural anomaly. Prognosis is poor if not treated. 75% survival for immune hydrops if treated with blood transfusion. Near 100% fatal if hydrops + fetal anomaly (excluding tachyarrhythmia).
59
Q

23.CPAM which is false:

  1. Commonly assoc with other anomalies
  2. Assoc with mediastinal shift
  3. Surgery is usually recommended for asymptomatic patients with Dx of CCAM made on prenatal US
  4. Can cause fetal hydrops
A
  1. Commonly assoc with other anomalies - F - 25% have associated structural anomalies, and fetal karyotype is indicated in these cases
  2. CPAM which is false: (TW)
  3. Commonly assoc with other anomalies - F - 25% have associated structural anomalies, and fetal karyotype is indicated in these cases
  4. Assoc with mediastinal shift - T - contralateral mediastinal shift in 90%
  5. Surgery is usually recommended for asymptomatic patients with Dx of CCAM made on prenatal US - T - UpToDate (Jan 2009) - early intervention is recommended to prevent the potential complications of recurrent infections and/or malignancy. In symptomatic patients immediate surgical intervention may be required.
  6. Can cause fetal hydrops - T - large CCAMs and shift of the mediastinum may lead to obstruction of the IVC and cardiac compression, resulting in development of hydrops from increased central venous pressure.
60
Q

26.Which is true regarding USS in pregnancy:

  1. FHR 85bpm normal at 5-8 weeks
  2. Post U.V commonest cause of hydronephrosis
  3. In TOF , the major abnormality is enlargement of right ventricle in utero
  4. Oligohydramnios in 1st trimester is of no clinical significance
  5. Low position of gestational sac carries a risk of abnormal pregnancy
A
  1. Low position of gestational sac carries a risk of abnormal pregnancy - T - the position of the gestational sac is important. IN a normal anteverted uterus, the sac should be situated in the upper uterine body, in a midposition between the uterine walls. A lower-lying GS may be normal, but is more suggestive of a miscarriage (either an impending spontaneous pregnancy loss, or one in progress). Cervical ectopic also to be considered (although in the 1st TM the internal Cx os may be difficult to identify, the insertion of the uterine arteries into the lateral margin of the uterus can define this region).
  2. Which is true regarding USS in pregnancy: (GC & Teacup)
  3. FHR 85bpm normal at 5-8 weeks F - At 5 to 6 weeks, the heart rate averages 100bpm, rising to 140-160 bpm by 8 weeks (US Requisites). Bradycardia considered <100bpm (or 90 bpm – SK). <6 weeks 100-115bpm, by 8 weeks 144-159bpm, >9 weeks 137-144bpm (blackbook).
  4. Posterior urethral valve commonest cause of hydronephrosis F - PUJ obstruction accounts for up to 2/3 of cases. Other causes of congenital hydropnephrosis: PUV (18%), ectopic ureterocoele, prune belly syndrome, utereral + VUJ obstruction, severe reflux, bladder neck obstruction…
  5. In TOF , the major abnormality is enlargement of right ventricle in utero - F - right and left heart pressures are almost equal in fetus. Best imaging clue is dilated aortic root overriding VSD.
  6. Oligohydramnios in 1st trimester is of no clinical significance - F - 1st TM olidgohydramnios is a poor prognostic finding. Potential value in comparing size of gestational sac to embryonic length. Subtracting CRL (mm) from MGSD (mm) - a value greater than 5mm is normal. When value is less than 5mm, almost all (94%) abort spontaneously. This measurement still needs more investigation.
  7. Low position of gestational sac carries a risk of abnormal pregnancy - T - the position of the gestational sac is important. IN a normal anteverted uterus, the sac should be situated in the upper uterine body, in a midposition between the uterine walls. A lower-lying GS may be normal, but is more suggestive of a miscarriage (either an impending spontaneous pregnancy loss, or one in progress). Cervical ectopic also to be considered (although in the 1st TM the internal Cx os may be difficult to identify, the insertion of the uterine arteries into the lateral margin of the uterus can define this region).
61
Q

27.Regarding obstetric ultrasound which is FALSE

  1. MCDA MZ twins are more common than DCDA MZ twins
  2. The majority of dichorionic twins are dizygotic
  3. The number of yolk sacs = number of amnions
  4. A single placental mass in a twin pregnancy excluded dichorionic pregnancy
A
  1. A single placental mass in a twin pregnancy excluded dichorionic pregnancy - F - placenta can fuse in DC (ie 2 placentas) MA (2 amniotic sacs) giving appearance of a single placenta.
  2. Regarding obstetric ultrasound which is FALSE (TW)
  3. MCDA MZ twins are more common than DCDA MZ twins - T - MZ-MCDA monozygotic 60%, MZ-DCDA 30%, , MCMA 10%.
    * LW: RP numbers similar.
    - -> DZ 70%. MZ 30%
    - -> MZ breakdown: MZ MCDA = 75%. MZ DCDA = 20%. MZ MCMA = 5% . MZ MCMA conjoined <1%
  4. The majority of dichorionic twins are dizygotic - T - DZ in 2/3rds, MZ in 1/3rd.
  5. The number of yolk sacs = number of amnions - T - but yolk sac easier to see cf amnion. May only see one initially (ie 2 GS). With early follow up, occasionally a second sac will be see if development is delayed
  6. A single placental mass in a twin pregnancy excluded dichorionic pregnancy - F - placenta can fuse in DC (ie 2 placentas) MA (2 amniotic sacs) giving appearance of a single placenta.
62
Q

28.The following statements are false regarding US findings in the first trimester pregnancy:

  1. A fetal heart rate of 100 beats per minute is normal at 5-8 weeks gestation
  2. The absence of a yolk sac on transvaginai US, when the gestation sac is equal than 8mm indicates pregnancy failure
  3. The absence of a double decidual sign is an indicator of abnormal pregnancy
  4. Oligohydramnios is of no clinical significance
  5. A low position of the gestation sac in the uterine cavity carries an increased risk abnormal pregnancy
A
  1. Oligohydramnios is of no clinical significance - F - 1st TM olidgohydramnios is a poor prognostic finding. Potential value in comparing size of gestational sac to embryonic length. Subtracting CRL (mm) from MGSD (mm) - a value greater than 5mm is normal. When value is less than 5mm, almost all (94%) abort spontaneously. This measurement still needs more investigation.
  2. The following statements are false regarding US findings in the first trimester pregnancy: (GC & TW)
  3. A fetal heart rate of 100 beats per minute is normal at 5-8 weeks gestation - T - At 5 to 6 weeks, the heart rate averages 100bpm, rising to 140-160 bpm by 8 weeks (US Requisites). Bradycardia considered <90bpm.
  4. The absence of a yolk sac on transvaginal US, when the gestation sac is greater than 8mm indicates pregnancy failure T - definite visualisation of yolk sac on TV scan at 5.5wks gestation, in GS with MSD of ≥ 10mm. For TA scan, should see yolk sac at 7wks gestation, in GS with MSD of ≥ 20mm.
  5. The absence of a double decidual sign is an indicator of abnormal pregnancy - T - after 5 weeks, DDS should be seen in 98%. Comprises 2 concentric hyperechoic rings surrounding a portion of the gestational sac (outer ring is decidua parietalis (decidua vera), inner ring is decidua capsularis (chorionic villi), sandwiched btn are the apposed endometrial walls). Callen p1027 pics
  6. Oligohydramnios is of no clinical significance - F - 1st TM olidgohydramnios is a poor prognostic finding. Potential value in comparing size of gestational sac to embryonic length. Subtracting CRL (mm) from MGSD (mm) - a value greater than 5mm is normal. When value is less than 5mm, almost all (94%) abort spontaneously. This measurement still needs more investigation.
  7. A low position of the gestation sac in the uterine cavity carries an increased risk abnormal pregnancy - T - the position of the gestational sac is important. In a normal anteverted uterus, the sac should be situated in the upper uterine body, in a midposition between the uterine walls. A lower-lying GS may be normal, but is more suggestive of a miscarriage (either an impending spontaneous pregnancy loss, or one in progress). Cervical ectopic also to be considered (although in the 1st TM the internal Cx os may be difficult to identify, the insertion of the uterine arteries into the lateral margin of the uterus can define this region).
63
Q

29.Regarding Cervical Cancer which is true:

  1. The MR signal of endocervical canal is low on T2 weighted imaging
  2. The MR signal of the endocervical canal is high on T1 weighted imaging
  3. FIGO IIIb stage indicates parametrial involvement
  4. Cervical cancer is low signal on T2 weighted imaging
  5. Cervical cancer demonstrates enhancement post gadolinium
A

5.Cervical cancer demonstrates enhancement post gadolinium T – early contrast enhancement on T1 fat sat (Dahnert).

Body MRI (Roth) – enhancement is variable but present. (SK)

  1. Regarding Cervical Cancer, which is true: (GC/SK)
  2. The MR signal of endocervical canal is low on T2 weighted imaging F – endocervical canal hyperintense, mid layer hypointense, outer layer intermediate T2 signal
  3. The MR signal of the endocervical canal is high on T1 weighted imaging F - should be low. On T1WI’s the whole uterus exhibits intermediate signal (SK).
  4. FIGO IIIb stage indicates parametrial involvement F - this is stage IIb (non surgical management). Parametrial invasion best seen on T1 as low signal spiculated areas of soft issue radiating from periphery of cervix.
  5. Cervical cancer is low signal on T2 weighted imaging F – hyperintense on T2 (relative to cervical fibrous stroma), regardless of histologic type. Size of tumour accurately depicted on T2 (rarely overestimated due to inflammation/oedema). Roth – the tumour is intermediate in T2 signal – brighter than the inner fibromuscular stroma & darker than normal cervical mucosa
  6. Cervical cancer demonstrates enhancement post gadolinium T – early contrast enhancement on T1 fat sat (Dahnert). Body MRI (Roth) – enhancement is variable but present. (SK)
64
Q

33.Regarding intrauterine ultrasound: t/f

  1. Choroid cysts strongly associated with trisomy 18
  2. Spina bifida the commonest cause of ventriculomegaly
  3. Duodenal atresia associated with Downs syndrome in 75% of cases
  4. Absence of renal cystic dysplasia makes Meckel-Gruber Syndrome unlikely
A

true 2, 4

  1. Regarding intrauterine ultrasound: (TW)
  2. Choroid cysts strongly associated with trisomy 18 - F - CPCs can be a normal variant in up to 3% of the population. It is also a soft marker for aneuploidy, particularly T18, and also weak association with T21.
  3. Spina bifida the commonest cause of ventriculomegaly - T - ventriculomegaly refers to mean diameter of lateral ventricular atrium of >/= 10cm. Can be due to obstruction (hydrocephalus), dysgenesis, or atrophy. In cases where obstruction is not visualised, ventriculomegaly is the more appropriate term. Spina bifida is most common cause of ventriculomegaly (30-60%, and 90% of SB have ventriculomegaly), however aqueductal stenosis is most common cause of fetal hydrocephalus (~40%).
  4. Duodenal atresia associated with Downs syndrome in 75% of cases - F - 30% of duodenal atresia have Down Syndrome. 3-5% of Downs syndrome have duodenal atresia (or other gastrointestinal abnormalities) UpToDate. Note that blackbook says 15% of T21 have DA.
  5. Absence of renal cystic dysplasia makes Meckel-Gruber Syndrome unlikely - T - Renal cystic dysplasia seen in 100% MGS. Encephalocele 60-80%, postaxial polydactyly 55-75%. MGS is autosomal recessive disorder, lethal malformation. Need to karyotype to exclude T13. (Blackbook).
65
Q
  1. Regarding neonatal anomalies, which is false:
  2. Omphalocele covered by a membrane
  3. Gastroschisis associated with bowel infartion
  4. Beckwith-Weidemann syndrome associated with omphalocele
  5. Bochdalek hernia has greater association with anomalies than Morgagni
A
  1. Bochdalek hernia has greater association with anomalies than Morgagni F - Morgagni often associated with congenital heart disease, bowel malrotation, chromosomal abnormality (Downs, Turners), mentral retardation, pericardial deficiency. Bochdaleck associated with herniated organs, more common on left (80%).
  2. Regarding neonatal anomalies, which is false: (GC)
  3. Omphalocele covered by a membrane T - failure of central fusion at the umbilical ring due to defective mesodermal growth causes incomplete closure of the abdominal wall and persistent herniation of the midgut. The abdominal viscera are contained in a translucent sac, which is composed of amnion, Wharton jelly, and peritoneum. The umbilical vessels radiate onto the wall of the sac. In 50% of cases, the liver, spleen, and ovaries or testes accompany the extruded midgut. High incidence of associated anomalies (chromosomal, GU, cardiac, NTD, IUGR, BWS, GI tract, limb-body wall deficiency, cystic hygroma).
  4. Gastroschisis associated with bowel infarction T - paramedian full-thickness abdominal fusion defect usually on right side of umbilical cord; may involve thorax; bowel is nonratated and lacks secondary fixation to dorsal abdominal wall. Gut complications before birth include bowel obstruction, peritonitis/perforation; after birth - malrotation, atresias, bowel necrosis, NEC, prolonged intestinal dysfunction, chronic short-gut syndrome.
  5. Beckwith-Weidemann syndrome associated with omphalocele T – “EMG” syndrome (exomphalos, macroglossia, gigantism). Common autosomal dominant overgrowth syndrome with reduced penetrance and variable expressivity related to short arm of chromosome 11 (that’s the short “p” one), sporadic in 85%. Omphalocoele seen in 76% of BWS babies; of all omphalocoeles, 5-10% are associated with BWS. Increased risk of Wilm’s > adrenocortical neoplasm > hepatoblastoma.
  6. Bochdalek hernia has greater association with anomalies than Morgagni F - Morgagni often associated with congenital heart disease, bowel malrotation, chromosomal abnormality (Downs, Turners), mentral retardation, pericardial deficiency. Bochdaleck associated with herniated organs, more common on left (80%).
66
Q

35.Regarding obstetric ultrasound: (T/F)

  1. Nuchal translucency is measured from 11 to 14 weeks
  2. Isolated choroid plexus cysts carry a high risk of Trisomy 18
  3. Vasa praevia is associated with fetal blood loss
  4. In the fetus, mild ureteric dilatation is a more significant finding than is mild dilatation of the renal pelvis
  5. A single placental mass in a twin pregnancy excludes dichorionic pregnancy
A

true 1, 3, 4

  1. Regarding obstetric ultrasound: (T/F) (TW)
  2. Nuchal translucency is measured from 11 to 14 weeks - T - NT is measured earlier in pregnancy than nuchal fold (layer of soft tissue identified immediately posterior to the occipital bone in the same scan plane as that used to identify the cerebellum and cisterna magna). NT measured from 11-14 wks, or CRL 45-84mm. (UTD says 36-84mm).
  3. Isolated choroid plexus cysts carry a high risk of Trisomy 18 - F - CPCs can be a normal variant in up to 3% of the population. It is also a soft marker for aneuploidy, particularly T18, and also weak association with T21. CPCs resolve in 3rd TM.
  4. Vasa praevia is associated with fetal blood loss - T - if vessels of a velamentous umbilical cord insertion of blood vessels extending between a succenturiate lobe and the main lobe of the placenta cross the internal os, this is called vasa previa. Can lead to significant fetal blood loss and fetal death at the time of rupture of membranes or delivery.
  5. In the fetus, mild ureteric dilatation is a more significant finding than is mild dilatation of the renal pelvis - T - PUJ obstruction is most common ngental malformation of urinary tract. Mild dilatation may be within normal limits (4mm before 33/40, 7mm (some use 10mm) after 33/40). Idiopathic fetal pyelectasis seen in 2-3% of prenatal cases. When ureters ureters or urinary bladder, or both are more dilated than the pelvocaliceal system this may represent early stage of blader outlet obstruction or end stafe of distal obstruction with dysplastic upper tracts (but more marked dilatation).
  6. A single placental mass in a twin pregnancy excludes dichorionic pregnancy - F - no because although there are 2 placentas they can fuse to one placenta
67
Q

36.Regarding obstetric ultrasound, which is false:

  1. Choroid plexus cysts associated with trisomy 18
  2. Vasa previa associated with fetal blood loss
  3. Mild ureteric dilatation more significant than mild renal pelvis dilatation alone
  4. Single placental mass excludes dichorionic pregnancy
A
  1. Single placental mass excludes dichorionic pregnancy F – placentas can fuse to form a single placenta which cannot be differentiated on US. Hence the importance of the early US examination / membrane assessment.
  2. Regarding obstetric ultrasound, which is false: (TW)
  3. Choroid plexus cysts associated with trisomy 18 - T - CPCs can be a normal variant in up to 3% of the population. It is also a soft marker for aneuploidy, particularly T18, and also weak association with T21. 1/3rd of T18 have CPCs (+ other anomalies).
  4. Vasa previa associated with fetal blood loss - T - if vessels of a velamentous umbilical cord insertion of blood vessels extending between a succenturiate lobe and the main lobe of the placenta cross the internal os, this is called vasa previa. Can lead to significant fetal blood loss and fetal death at the time of rupture of membranes or delivery.
  5. Mild ureteric dilatation more significant than mild renal pelvis dilatation alone - T - mild renal pelvis dilatation may be within normal limits, or idiopathic pyelectasis (2-3% of prenatal cases). Dilated ureters may reflect a distal obstruction.
  6. Single placental mass excludes dichorionic pregnancy F – placentas can fuse to form a single placenta which cannot be differentiated on US. Hence the importance of the early US examination / membrane assessment.
68
Q
  1. Downs syndrome associated with (False):
  2. Duodenal atresia
  3. Decreased acetabular angle
  4. Increased iliac angle
  5. Short little finger
  6. Addition ossification centre in the manubrium
A
  1. Increased iliac angle - F – decreased iliac angle and index in 70-80%. This is very ambiguous. It talking about iliac angle as measured from Hilgenreiners line, Downs is reduced (around 44 degrees, vs normal of 55 degrees). However, if iliac angle (ie angle of the iliac wings as measured on antenatal imaging) this angle is increased (75 degrees in DS, vs 60 degrees in normal fetus). See Radiology 1999. Dahnert mentions decreased iliac angle in one column, and increased iliac angle on other column.
  2. Downs syndrome (not) associated with: (TW)
  3. Duodenal atresia - T - double bubble. Assoc with duodenal atresia / sternosis / annular pancreas.
  4. Decreased acetabular angle - T – flattened acetabular roof therefore decreased acetabular angle
  5. Increased iliac angle - F – decreased iliac angle and index in 70-80%. This is very ambiguous. It talking about iliac angle as measured from Hilgenreiners line, Downs is reduced (around 44 degrees, vs normal of 55 degrees). However, if iliac angle (ie angle of the iliac wings as measured on antenatal imaging) this angle is increased (75 degrees in DS, vs 60 degrees in normal fetus). See Radiology 1999. Dahnert mentions decreased iliac angle in one column, and increased iliac angle on other column.
  6. Short little finger - T - hypoplastic and triangular middle and distal phalanges of 5th finger - acromicria.
  7. Addition ossification centre in the manubrium - T – hypersegmentation of manubrium = 2-3 ossification centers. It is not uncommon for the normal manubrium to develop from more than one centre of ossification, although there is generally one major centre and a variable number of smaller centers. These centers will quickly coalesce to form one major centre. 90% of Downs have 2 centers, one above the other.
69
Q
  1. Endometrial thickening greater than 15mm can be caused by:
  2. Amenorrhoea due to weight loss
  3. Turners
  4. Tamoxifen
  5. Adenomyosis
A
  1. Tamoxifen T - non steroidal anti-oestrogen in breast, acts as a weak oestrogen agonist causing proliferative effects on the endometrium. Increased prevalence of endometrial hyperplasia, polyps and carcinoma. 50% of women treated with tamoxifen will develop endometrial abnormalities within 6-36 months.
  2. Endometrial thickening greater than 15mm can be caused by: (GC)
  3. Amenorrhoea due to weight loss F - causes endometrial atrophy. Amenorrhoea classified as primary or secondary. Primary causes broken into: hypothalamic, pituitary, ovarian, functional (eg. anorexia, excessive stress or exercise).
  4. Turners F - included in ‘ovarian’ causes of amenorrhoea. Due to nondisjunction of sex chromosomes. Complete monosomy (45XO) accounts for 70%; nonvisualised/streaky ovaries and a prepubertal uterus. Partial monosomy (structurally altered second X chromosome), and mosaicism (XO plus another sex karyotype) are rare; ovaries may be normal in these types.
  5. Tamoxifen T - non steroidal anti-oestrogen in breast, acts as a weak oestrogen agonist causing proliferative effects on the endometrium. Increased prevalence of endometrial hyperplasia, polyps and carcinoma. 50% of women treated with tamoxifen will develop endometrial abnormalities within 6-36 months.
  6. Adenomyosis F - causes focal/diffuse myometrial heterogeneity, widened junctional zone with little distrotion of endometrial stripe.
70
Q

40.Features of Turners Syndrome include:

  1. Increased carpal index
  2. Hypoplastic 4th metacarpal
  3. Cubitus varus
  4. Delayed skeletal maturation
  5. PCKD
A
  1. Hypoplastic 4th metacarpal T - relative shortening of 4th and 5th metacarpals.
  2. Features of Turners Syndrome include: (GC)
  3. Increased carpal index F - narrowed carpal angle (scapho-lunate-triquetrum angle <177 degrees)
  4. Hypoplastic 4th metacarpal T - relative shortening of 4th and 5th metacarpals.
  5. Cubitus varus F – cubitus valgus: bilateral radial tilt of articular surface of trochlea
  6. Delayed skeletal maturation F - Dahnert: Normal skeletal maturation with growth arrest at skeletal age of 15yrs; delayed fusion of epiphyses > age 20 yrs. Osteoporosis during/after 2nd decade due to gonadal hormone deficiency. Nelsons Paediatrics: Short stature, the cardinal finding in all girls with Turner syndrome, may be present with minimal other clinical manifestations. The growth deceleration (cf. “delay”) begins in infancy and young childhood, gets progressively more pronounced in later childhood and adolescence, and results in significant adult short stature.
  7. PCKD F - approx 30-50% have a renal malformation, most commonly horseshoe kidney, followed by abnormal vascular supply. Due to nondisjunction of sex chromosomes as (1) complete monosomy (45XO), (2) partial monosomy (structurally altered second X chromosome), (3) mosaicism (XO plus another sex karyotype). 1 in 3000-5000 live births. Sexual infantilism (primary amenorrhoea, absent secondary sex characteristics), short stature, webbed neck with low posterior hairline, broad chest with widely spaced nipples, high palate, thyromegaly… Associations: horseshoe kidney, coarctation, bicuspid aortic valve. [Dahnert, Robbins]
71
Q

42.Regarding pleural effusion on antenatal scan: which is false

  1. Most commonly chylothorax
  2. Not necessarily indicative of a poor outcome
  3. Associated with most cases of CCAM
  4. May be seen in Turner’s syndrome
A
  1. Associated with most cases of CCAM F - polyhydramnios (25-75%), ascites (60-70%), hydrops in 33-80%. All indicate a poor outcome. In many fetuses, the CAM becomes smaller and may occasionally disappear by birth. [Dahnert]
  2. Regarding pleural effusion on antenatal scan: (GC)
  3. Most commonly chylothorax T
  4. Not necessarily indicative of a poor outcome T - see below.
  5. Associated with most cases of CCAM F - polyhydramnios (25-75%), ascites (60-70%), hydrops in 33-80%. All indicate a poor outcome. In many fetuses, the CAM becomes smaller and may occasionally disappear by birth. [Dahnert]
  6. May be seen in Turner’s syndrome T - chromosomal anomalies: 45XO and trisomy 21.

The clinical course of primary fetal hydrothorax is unpredictable. Whereas smaller unilateral effusions might remain stable or even regress, this is rarely the case with larger collections. Bilateral effusions, hydrops, preterm delivery and the lack of antenatal therapy are all associated with poor outcome. Once structural and chromosomal anomalies have been excluded, optimal management depends on gestational age, rate of progression, the development of hydrops and associated maternal symptoms. For very large effusions with mediastinal shift, hydrops and/or hydramnios, or when there is rapid enlargement of the effusion, fetal intervention is warranted. Survival can be maximized by pleuroamniotic shunting, which can reverse hydrops and hydramnios and prevent pulmonary hypoplasia. Pleuroamniotic shunting can also be used for the treatment of other large cystic lung lesions, such as a macrocystic CCAM or bronchopulmonary sequestration, especially when associated with hydrops. [Best Pract Res Clin O&G 2008].

72
Q
  1. Fragile X
  2. mental retardation
  3. mandible long
  4. hyperextensable lax joints
A
  1. mental retardation
  2. hyperextensable lax joints

Fragile X is the most common inherited form of mental retardation. Expansion of the CGG trinucleotide segment of the fragile X mental retardation (FMR-1) gene. Males can get autism, mental impairment, seizures, big forehead, mandibular prognathism, macrocephaly, big balls/nuts/testicles, flexible joints, flat feet, mitral valve prolapse, and are slightly short (height). Females have milder abnormalities (as have 2x X-chromosomes).

73
Q
  1. Which of the following is not a sex cord/stromal tumour (of ovary)
  2. Fibroma
  3. Granulosa-theca cell tumor
  4. Sertoli-leydig cell tumor
  5. Yolk Sac tumor
A

4.Yolk Sac tumor

74
Q
  1. Who is at the greatest risk of hydatiform mole
  2. Israel
  3. Sweden
  4. Netherlands
  5. USA
  6. Indonesia
A

5.Indonesia

75
Q
  1. Diagnosis of trophoblastic disease is dependent on
  2. curettage
  3. histopathology absence of syncitiotrophoblasts
A

1.curettage

76
Q

47.Female, primary amenorrhoea, short stature. Pelvic ultrasound - no ovaries seen. Most likely diagnosis:

  1. Female pseudohermaphodism
  2. Turners syndrome
  3. Klinefelter’s syndrome
  4. True hermaphroditism
A
  1. Turners syndrome T - Due to nondisjunction of sex chromosomes as (1) complete monosomy (45XO), (2) partial monosomy (structurally altered second X chromosome), (3) mosaicism (XO plus another sex karyotype). 1 in 3000-5000 live births. Sexual infantilism (primary amenorrhoea, absent secondary sex characteristics), short stature, webbed neck with low posterior hairline, broad chest with widely spaced nipples etc. US: nonvisualised or “streak” ovaries (complete monosomy), may be normal in mosaics. Associations: horseshoe kidney, coarctation, bicuspid aortic valve.
  2. Female, primary amenorrhoea, short stature. Pelvic ultrasound - no ovaries seen. Most likely diagnosis: (GC)
  3. Female pseudohermaphroditism F - 46XX, caused by exposure to excessive androgens in 1st trimester due to (1) congenital androgenital syndrome, (2) maternal drug ingestion, (3) masculinising ovarian tumour. Masculinised external genitalia, normal ovaires/tubes/uterus/vagina, enlarged adrenal glands (hyperplasia), no testicular tissue or internal wolffian duct derivatives.
  4. Turners syndrome T - Due to nondisjunction of sex chromosomes as (1) complete monosomy (45XO), (2) partial monosomy (structurally altered second X chromosome), (3) mosaicism (XO plus another sex karyotype). 1 in 3000-5000 live births. Sexual infantilism (primary amenorrhoea, absent secondary sex characteristics), short stature, webbed neck with low posterior hairline, broad chest with widely spaced nipples etc. US: nonvisualised or “streak” ovaries (complete monosomy), may be normal in mosaics. Associations: horseshoe kidney, coarctation, bicuspid aortic valve.
  5. Klinefelter’s syndrome F - 47XXY, 1 in 750 live births. Testicular atrophy with small/absent testes and eunuchoid constitution.
  6. True hermaphroditism F - have both ovarian and testicular tissues in the gonads. Ambiguous external genitalia.
77
Q
  1. 38yo with bilateral enlarged ovaries and marked endometrial thickening. ?cause
  2. Polycystic ovaries
  3. Granulosa theca ovarian tumour
  4. Endometrial carcinoma with bilateral ovarian metastases
A
  1. Polycystic ovaries - T - bilaterally enlarged ovaries >10cc with >12 follicles (2-9mm). Endometrial hyperplasia/cancer/polyps may occur due to unopposed chronic E2 stimulation.
  2. 38 yo with bilateral enlarged ovaries and marked endometrial thickening. Most likely cause: (GC & TW)
  3. Polycystic ovaries - T - bilaterally enlarged ovaries >10cc with >12 follicles (2-9mm). Endometrial hyperplasia/cancer/polyps may occur due to unopposed chronic E2 stimulation.
  4. Granulosa theca ovarian tumour F - most common hormone-active oestrogenic tumour of ovary. Typically unilateral, but 5% are bilateral. 45% occur in women of reproductive age, and 50% postmenopausal. Oestrogenic effects: uterine enlargement, endometrial thickening/haemorrhage, endometrial hyperplasia/polyps/Ca.
  5. Endometrial carcinoma with bilateral ovarian metastases - F - although endometrial Ca is most common malignancy, ovarian lesions usually direct spread, not metastatic. GC: Two possible routes for spread: via fallopian tube to surface of ovary, or via lymphatics to the inside of the ovary (uncommon, but would cause enlargement). [Ov mets in ECa, Gyn Onc 1998]
  6. Oral contraceptive pill F - downregulation of ovaries and endometrium.
78
Q

49.Which is not typical of endometrial carcinoma?

  1. focal mass
  2. diffuse thickening of the uterus
  3. tubal to peritoneal spread
  4. if SCC then it is likely to be cervical carcinoma spreading to the uterus
A

*LW to check: differs from path answer
*LW: ROBBINS:
spread generally occurs by direct myometrial invasion with eventual extension to peri uterine structures by direct continuity. Spread to nodes eventually occurs, , followed by mets to lungs liver bones.
In papillary serous carcinoma sub type, approx 20% , has extensive peritoneal disease suggesting peri tubal or lymphatic spread, rather than direct invasion.
FIGO staging follows the more commen direct invasion extension pathway, so I would favour this option of tubal to peritoneal spread is not typical for endometrial carcinoma (i.e only occurs in approx 20% of cases which are the serous sub type)

  1. tubal to peritoneal spread Transmural or transtubal tumour spread into the peritoneal cavity occurs with more aggressive cancers (ie. serous Ca). These are less common (5-10%), versus 75-80% for endometrioid adenocarcinoma. ‘Typical’ disease progression beyond the uterus occurs through lymphatic pathways into pelvic and abdominal lymph nodes. The cancer eventually may metastasize haematogenously to the lungs > liver > brain, or bones.
  2. Which is not typical of endometrial carcinoma? (GC)
  3. focal mass T - usually arises from the glandular component of the endometrium in the upper uterus. It may grow in a focal circumscribed pattern presenting as a friable mass protruding into the endometrial cavity. It can occasionally arise within an endometrial polyp.
    * LW agree
  4. diffuse thickening of the uterus T - may be diffuse, involving multiple regions of the endometrium or the whole endometrial surface.
    * LW: agree

3.tubal to peritoneal spread Transmural or transtubal tumour spread into the peritoneal cavity occurs with more aggressive cancers (ie. serous Ca). These are less common (5-10%), versus 75-80% for endometrioid adenocarcinoma. ‘Typical’ disease progression beyond the uterus occurs through lymphatic pathways into pelvic and abdominal lymph nodes. The cancer eventually may metastasize haematogenously to the lungs > liver > brain, or bones.

4.if SCC then it is likely to be cervical carcinoma spreading to the uterus T - 85% of endometrial carcinoma are adenoCa, and 60% of these are endometrioid Ca. Squamous metaplasia is present in 20% of endometrioid adenocarcinomas, but these are foci of benign squamous differentiation (ie. adenocanthoma). The malignant subtype is adenosquamous Ca (endometrioid with >10% malignant squamous component).
*LW agrees with this statement.
Conversely, 75% of cervical Ca are SCC (followed by adenosquamous Ca, 20%; and small cell neuroendocrine Ca, <5%). Most common mode of spread is direct extension to lower uterine segment and vagina.

*LW this argument differs to the path MCQ - need to check**

79
Q
  1. Placental ultrasound.
  2. Once a placenta previa has been demonstrated via abdominal ultrasound in the third trimester this should be confirmed by a transvaginal scan.
  3. The presence of an intra-uterine sac excludes an ectopic.
A

1.Once a placenta previa has been demonstrated via abdominal ultrasound in the third trimester this should be confirmed by a transvaginal scan.

80
Q
  1. Ultrasound of the placenta. (T/F)
  2. A placenta measuring less than 3 cm. indicates IUGR
  3. Greater than 10 cm. hypoechoic collections within the placenta suggest an abruption.
  4. Separation of the chorionic plate from the placental margin suggests abruption.
  5. A normal yolk sac is a poor prognostic indicator.
  6. A low lying sac suggests an inevitable abortion.
A

true 3, 5

  1. Ultrasound of the placenta.(T/F) (TW)
  2. A placenta measuring less than 3 cm indicates IUGR - F- Midportion of a normal placenta should always remain thicker than 2cm. Abnormal placental thinning cn be caused by maternal systemic vascular problems particularly associated with microinfarctions in cases of IDDM and collagen vascular diseases and in severe HTN. Thinned placenta can put fetus at risk for growth restriction. False positives: marked polyhydramnios where placenta stretched and thinned along the enlarged uterus.
  3. Greater than 10 cm hypoechoic collections within the placenta suggest an abruption. - F - placental abruption - categorized by location as retroplacental, marginal, pre-placental. Not within placenta. Marginal abruption is most common. Abruption is a clinical Dx and US may be negative.
  4. Separation of the chorionic plate from the placental margin suggests abruption - T - marginal collection (at the placental margin), preplacental collection under the chorionic plate (between placenta and amniotic fluid), and “jello-like” movement of the chorionic plate with fetal activity are features of abruption. Raised edge of placenta seen in 50% cases.
  5. A normal yolk sac is a poor prognostic indicator - F
  6. A low lying sac suggests an inevitable abortion - T - the position of the gestational sac is important. In a normal anteverted uterus, the sac should be situated in the upper uterine body, in a midposition between the uterine walls. A lower-lying GS may be normal, but is more suggestive of a miscarriage (either an impending spontaneous pregnancy loss, or one in progress).
    * *LJS or ddx of scar ectopic
81
Q

53.Foetal anomalies identifiable in utero: (T/F)

  1. Gastroschisis is essentially an isolated entity
  2. Pseudo omphalocoele may be seen in oligohydramnios
  3. Diaphragmatic herniations usually occur in isolation
  4. Achondroplasia is the commonest non lethal skeletal dysplasia
  5. 60% of patients with duodenal atresia have trisomy 21
A

1,2,4 true

  1. Foetal anomalies identifiable in utero: (T/F) (GC)
  2. Gastroschisis is essentially an isolated entity T - associated anomalies in 5% (vs 45-88% in omphalocoele). These include intestinal atresia/stenosis (small size of opening leads to compression or torsion of vessels); ectopia cordis (rare).
  3. Pseudo omphalocoele may be seen in oligohydramnios T - (1) Deformation of fetal abdomen by transducer pressure coupled with an oblique scan orientation; (2) physiologic herniation of midgut into umbilical cord btn 8-12 wks gestation.
  4. Diaphragmatic herniations usually occur in isolation F - Associated anomalies in 20% of liveborn and 90% of stillborn babies. These include NTD/spinal defects, cardiovascular, GIT (esp. malrotation, oral cleft, omphalocoele), chromosomal (trisomy 18+21), IUGR. Right-sided hernias are frequently fatal (L5-9:R1). However, the most common type are Bochdaleck (85-90%), and these are less likely to be associated with other anomalies cf. Morgagni hernias.
  5. Achondroplasia is the commonest non lethal skeletal dysplasia T - heterozygous achondroplasia (1 in 30,000 births). Homozygous is lethal in neonatal period (from respiratory failure).
  6. 60% of patients with duodenal atresia have trisomy 21 F - 25% of fetuses with duo atresia have Down’s
82
Q
  1. The following are usually seen in the prune belly syndrome: (T/F)
  2. Megacystis
  3. Cryptorchidism
  4. Polyhydramnios
  5. Ureterectasis
  6. Hypertrophied anterior abdominal wall musculature
A

true 1,2, , 4

  1. The following are usually seen in Prune belly syndrome: (T/F) (GC)
  2. Megacystis T - large distended bladder, thickened wall without trabeculations (due to replacement by fibroblasts and collagen), intramural calcifications, widely patent bladder neck. Two main groups: (1) severe urethral obstruction [eg. urethral atresia], (2) functional abnormality of bladder emptying [more common]. Group 1 has a high association with other anomalies; very poor prognosis (due to renal failure +/- pulmonary insufficiency).
  3. Cryptorchidism T - essential component. Triad of abdominal wall muscle deficiency, non obstructed hydroureteronephrosis, and bilateral cryptorchidism. Increased risk of malignant degeneration.
  4. Polyhydramnios F - normal or decreased AFI
  5. Ureterectasis T - massively dilated tortuous elongated ureters affecting the lower third more profoundly (hallmark).
  6. Hypertrophied anterior abdominal wall musculature F - wrinkled flaccid appearance of hypotonic abdominal wall with bulging flanks (agenesis/hypoplasia of ventral and lateral musculature).
83
Q

55.Sonography of multiple gestations (T/F)

  1. 80% of twin pregnancies are dichorionic
  2. Intermingling of the 2 umbilical cords allows confident diagnosis of mono amniocity
  3. Death of one twin in the second trimester may lead to foetus papyraceus
  4. Dichorionic twins may suffer from twin to twin transfusion
  5. Mono chorionic, mono amniotic twins have a 10 % mortality
A

true 1, 2, 3

  1. Sonography of multiple gestations: (T/F) (TW)
  2. 80% of twin pregnancies are dichorionic - T - 1:85 births are twins. Of these 1/3 are MZ, 2/3 DZ. All the DZ are DCDA (ie ~66%). Of the MZ - 30% are DCDA, 69-80% are MCDA, and 1% are MCMA. Summatively the total of DCDA’s is near 80%.
  3. Intermingling of the 2 umbilical cords allows confident diagnosis of monoamniocity - T - only definitive postiive sonographic sign of monoamnionicity as per Dahnert. Conjoined twins would also indicate MA.
  4. Death of one twin in the second trimester may lead to foetus papyraceus - T - flattened, mummified fetus assoc with a viable twin or multiple gestation. Intrauterine fetal demise of a twin after 8 weeks with retention of the fetus for minimum of 10wks results in mechanical compression of the small fetus such that it resembles parchment paper. Prior to 8 weeks of gestational age, the only evidence of an IU death of a twin may be a cyst on the fetal surface of the surviving twin.
  5. Dichorionic twins may suffer from twin to twin transfusion - F - TTTS is exclusive to monochorionic pregnancies (MCMA, MCDA). Complicates 10-20% of monochorionic pregnancies.
  6. Monochorionic, monoamniotic twins have a 10 % mortality - F - 54% perinatal mortality rate (prematurity, growth restriction, anomalies, cord accidents, vascular anastomoses, TTTS)
    * AJL - This seems very high. Found a paper that says: the overall perinatal mortality rate of MCMA was 14.7% (17/116), ranging from 69.2% at < 30 weeks to 4.5% at ≥ 33 weeks’ gestation.
    https: //pubmed.ncbi.nlm.nih.gov/29900612/

Complications of twin pregnancy:preterm delivery is 5x that of singleton rate.DCDA (developing embryo split at 3d) perinatal mortality 10%.MCDA (split 4-8d). MC twins have 3-5x perinatal morbidity and mortality relative toDC twins, and this increases again (8x) in chemically induced (IVF). MCMA (split 8-13d). 54% perinatal mortality.Congjoined (split >13d). 75% perinatal mortality.2727